MBE simulated exams Flashcards

1
Q

<p>A contractor gave the low bid for some electrical repairs to a homeowner’s house. Based on this bid, the contractor and the homeowner entered into a contract stating that the contractor would perform the electrical repairs for $6,000. Before beginning work on the project, the contractor notified the homeowner that he would lose money on the job at that price, and would not proceed with the work unless the homeowner would agree to increase the price to $9,000. The homeowner thereupon, without notifying the contractor, entered into a contract with an electrician to make the repairs for $7,500, which was the fair market cost of the work to be done. The electrician finished the house on schedule and then showed the homeowner that he (the electrician) had spent $8,500 on the job. The homeowner thereupon paid the electrician the full balance of their contract price plus an additional $1,000, so that the electrician would not lose money on the job.

In a contract action by the homeowner against the contractor, how much will the homeowner recover?

A The difference between the fair market cost of the repairs and the contractor’s original contract price.

B $3,000, the difference between the contractor’s original contract price and the amount the contractor demanded.

C $2,500, the difference between the contractor’s original contract price and the total amount the homeowner paid the electrician for the repairs.

D $1,500, the difference between the contractor’s original contract price and the electrician’s contract price.</p>

A

<p>The issue here is the mitigation of damages so D

The homeowner can recover $1,500, the difference between the contractor’s contract price and the contract price of the substitute performance. Here, while the homeowner actually paid $2,500 more than the contractor’s contract price to have the house repaired, he was obligated to pay only $1,500 more because the electrician had a legal duty to make the repairs to the house for his contract price and no more. The homeowner will not be able to recover the $1,000 difference because he has a duty to mitigate damages, and paying more than he was actually obligated to pay breaches the duty. (A) and (B) are incorrect because they do not apply the proper measure of damages formula.</p>

How well did you know this?
1
Not at all
2
3
4
5
Perfectly
2
Q

<p>A delivery company employed several messengers to deliver packages by car to nearby towns. The company also allowed some employees to use company cars for personal use from time to time. A clerical employee had her car in a body shop because she had run a red light and been broadsided by another vehicle. This was the second time she had run a light and been hit. She borrowed a company car for the weekend and was using it to do some grocery shopping. The employee negligently went through a red light and crossed the path of a rented van. The man driving the van swerved to avoid the employee and struck a light post and several parked cars, severely damaging the van. At the time of the accident, the driver of the van was exceeding the posted speed limit; he would have been able to avoid hitting the light post and the cars had he been going the proper speed.

The leasing company that had rented the van to the driver brought a lawsuit against the delivery company employee and the delivery company. The jurisdiction retains traditional contributory negligence rules.

If the delivery company prevails in the lawsuit, what is the most likely reason?

A The delivery company had no reason to know that its employee had a poor driving record.

B The driver of the van had the last clear chance to avoid the accident.

C The driver of the van exceeded the posted speed limit.

D The employee had agreed to assume all liability when she borrowed the delivery company’s car.</p>

A

<p>If the delivery company prevails, it will be because it entrusted its car to its employee without having reason to know that she had a poor driving record. In the absence of negligence on the delivery company’s part, it will not be liable for its employee’s negligent driving either as her employer or as the owner of the automobile she was driving.

An employer will be vicariously liable for tortious acts committed by its employee only if the tortious acts occur within the scope of the employment relationship. Here, although the employee was using the company car, she was not conducting any business for the delivery company. Her use of the car to go grocery shopping was a personal errand outside the scope of her employment for which the delivery company is not vicariously liable.

The delivery company is also not vicariously liable for permitting its employee to drive its car—the general rule absent a statute to the contrary is that an automobile owner is not vicariously liable for the tortious conduct of another driving the owner’s automobile.

However, the owner may be liable for its own negligence in entrusting the car to a particular driver. If the delivery company knew or should have known that its employee had a poor driving record, its furnishing the employee with a car would constitute a breach of its duty to other drivers. However, if it is determined that the delivery company had no reason to know of the employee’s poor driving record, it will not be liable.

(B) is incorrect because the fact that the driver of the van had the last opportunity to avoid the accident is irrelevant to the leasing company’s right to recover from the delivery company. The doctrine of last clear chance does not apply because it is essentially plaintiff’s rebuttal against the defense of contributory negligence; it would not be raised by the delivery company as a defense (i.e., if the delivery company asserted that the driver of the van was contributorily negligent, the leasing company could rebut by asserting that the delivery company employee had the last clear chance to avoid the accident).</p>

How well did you know this?
1
Not at all
2
3
4
5
Perfectly
3
Q

<p>The federal Fair Opportunity Act provided that an employer whose products are in any way used by or sold to the federal government must meet certain very specific standards for the hiring of women in traditional male jobs. One city has since enacted an ordinance requiring that any employer doing business with the city have a workforce consonant with the gender composition of the population of the city. A private janitorial service located in the city does contract cleaning for both the city jail and some local federal government offices. The gender makeup of the company’s workforce violates the city’s ordinance but is in compliance with the federal Fair Opportunity Act. The janitorial service brings an action in state court to enjoin enforcement of the city ordinance. It argues that the local rule is invalid since it conflicts with the federal statute by creating more stringent standards.

How should the trial court rule?

A There is no conflict, because Congress intended that the Fair Opportunity Act apply only to employers who dealt exclusively with the federal government.

B There is no conflict, because the city is permitted to impose more strict requirements to deal with a local problem than those established by the federal government.

C The federal act preempts the local ordinance and thus the latter cannot be enforced.

D The federal act preempts the local ordinance only insofar as it attempts to regulate employers who do business with the federal government, so the ordinance may not be enforced only as to the janitorial company, but is otherwise valid.</p>

A

<p>There is no conflict between the two provisions. If the federal legislation does not address the issue of preemption, and a problem is uniquely local, the local government is permitted to enact rules more strict than the federal standards. The Court has made it clear that for preemption, either the federal statute must expressly preempt state measures, or it must be clear that the state law is impliedly preempted by the federal measure, either because the state law poses a conflict that makes compliance with both measures impossible, it prevents achievement of the federal objective, or the federal law “occupies the field” being regulated. Because the facts do not indicate that the federal measure expressly preempts state regulation, and because none of the implied preemption conditions are met</p>

How well did you know this?
1
Not at all
2
3
4
5
Perfectly
4
Q

<p>The plaintiff, an electrical contractor, sued the defendant homeowner for refusal to pay for extensive wiring repairs performed on his home by the plaintiff’s employee. The plaintiff called the employee to the stand. The employee, under oath, testified that he did not perform any work at the defendant’s home. The employee also denied writing a letter to a friend telling the friend that the employee was going to do electrical work on the home. Without releasing the employee as a witness, the plaintiff offers into evidence the letter written by the employee to his friend.

If the employee’s letter to his friend is properly authenticated, should the court admit the letter?

A Yes, for impeachment purposes only.

B Yes, as both substantive and impeachment evidence.

C No, because a party may not impeach his own witness.

D No, because it is inadmissible hearsay.</p>

A

<p>B because you can impeach with prior inconsistent statement and as substantive because it is a statement of present sense impression.

The letter is admissible as substantive evidence as well as for impeachment purposes. For the purpose of impeaching the credibility of a witness, a party may show that the witness has, on another occasion, made statements that are inconsistent with some material part of his present testimony. This may be done by first questioning the witness as to the prior inconsistent statement that he has made. If the witness denies having made the statement or fails to remember it, the making of the statement may be proved by extrinsic evidence. A proper foundation must be laid by giving the witness an opportunity to explain or deny the statement, and it must be relevant to some issue in the case. Here, the plaintiff’s employee has denied that he wrote the letter to his friend. The plaintiff can then impeach the employee by offering the letter into evidence. Because the employee has not been released as a witness, he will have an opportunity to explain or deny the statement, and it is relevant to whether any work was done at the home. Because prior inconsistent statements are generally hearsay, they often are admissible only for purposes of impeachment. The Federal Rules do categorize a testifying witness’s prior inconsistent statement as nonhearsay if it was made under penalty of perjury at a prior trial, hearing, or proceeding, or in a deposition.

Here, of course, the employee’s letter to the friend was not made under oath, so it is hearsay. However, it is still admissible as substantive evidence because it falls within an exception to the hearsay rule. Under Rule 803(3), a statement of a declarant’s then-existing state of mind is admissible as a basis for a circumstantial inference that the declarant acted in accordance with his state of mind. [See also Mutual Life Insurance Co. v. Hillmon (1892)]

The employee’s statement that he was going to do electrical work on the home is admissible as circumstantial evidence tending to show that he followed through with his plans and did the electrical work, which is what the statement is being offered to establish. In this case, therefore, the letter should be admissible as both substantive and impeachment evidence, making (B) correct and (A) incorrect. (C) is incorrect because the Federal Rules provide that the credibility of a witness may be attacked by any party, including the party calling him. [Fed. R. Evid. 607] (D) is incorrect. The letter is hearsay because it is being offered to prove the truth of the matter asserted—that the employee was going to do electrical work on the home—as a basis for inferring that the employee did do the work. Additionally, it is not categorized as nonhearsay under the Federal Rules because it was not made under oath. However, as discussed above, it falls within the “present state of mind” exception to the hearsay rule.</p>

How well did you know this?
1
Not at all
2
3
4
5
Perfectly
5
Q

<p>A chef agreed in writing to lease a restaurant from the owner of the property. The term of the tenancy was two years, and rent was payable in monthly installments at the beginning of each month. At the end of the second year, there had been no discussions between the chef and the owner regarding renewal or termination. The chef did not vacate the premises at the end of the term; instead, she sent a check for the next month’s rent to the owner. The owner cashed the check after the term had expired but informed the chef that his acceptance of the check did not mean that he was going to renew the lease or let the chef stay. At the end of that month, the owner seeks advice on whether he can evict the chef.

How should the owner be advised to proceed?

Press Enter or Space to submit the answer

A The owner must give the chef a full 30 days’ notice before beginning eviction proceedings because a month-to-month periodic tenancy has been created.

Correct
B The owner may begin eviction proceedings as soon as the additional month has expired.

Incorrect
C The owner may not evict the chef for 11 months and must give six months’ notice before beginning eviction proceedings because a year-to-year periodic tenancy has been created.

D The owner may not evict the chef for 11 months but need not give any notice prior to eviction because a tenancy for years for a term of one year has been created.</p>

A

<p>Landlord keeps rent for one month, be he did not decide to create a periodic tenancy.

The owner may begin eviction proceedings at any time. When a tenant continues in possession after the termination of her right to possession, the landlord has two choices of action: He may treat the hold-over tenant as a trespasser and evict her under an unlawful detainer statute, or he may, in his sole discretion, bind the tenant to a new periodic tenancy, in which case the terms and conditions of the expired tenancy apply to the new tenancy. Here, while the owner accepted the check sent by the chef, he informed her that he was not electing to bind her to a new tenancy. The owner may keep the check because he is entitled to rent for the month that the chef was a hold-over tenant, but at the end of that month he has the right to evict her because no periodic tenancy was created and the chef’s right to possession has terminated.

(A) is incorrect because the owner did not elect to create a periodic tenancy. Furthermore, had he done so, the tenancy would have been a year-to-year tenancy rather than a month-to-month tenancy because it is a commercial lease for more than one year, rather than a residential lease.

(C) is incorrect because, as discussed above, the owner did not elect to create a periodic tenancy when the chef held over.</p>

How well did you know this?
1
Not at all
2
3
4
5
Perfectly
6
Q

The plaintiff sued the defendant, who had constructed the plaintiff’s house, for breach of warranty of habitability. At trial, in cross-examination of the plaintiff, the defendant’s attorney asked whether the plaintiff had sued another contractor 30 years earlier, claiming similar defects in another house built for the plaintiff. The question was not objected to and the plaintiff answered that she had had some “water problems” with the first house she ever purchased, but no suit was filed. The defendant then called as a witness the contractor of 30 years earlier to testify that the plaintiff had brought suit against him for defects in the earlier house, many of which were like those now claimed to be found in the home the defendant built, but that the case was settled without trial.

Should the trial court rule that the witness’s offered testimony is admissible?
A Yes, as proper impeachment because the plaintiff will have an opportunity to explain or deny the witness’s statement.
B Yes, because the plaintiff failed to object to the defendant’s questions on cross-examination relative to the prior suit.
C No, because the best evidence of the former suit is the court record.
D No, because its probative value is substantially outweighed by the danger that it will confuse the issues and waste time.

A

The trial court should rule the witness’s testimony inadmissible because its probative value is substantially outweighed by the danger that it will confuse the issues and waste time. <strong>Where a witness makes a statement not directly relevant to the issues in the case, the rule against impeachment (other than by cross-examination) on a collateral matter applies to bar the opponent from proving the statement untrue either by extrinsic </strong>contradictory facts or by a prior inconsistent statement. The purpose of the rule is to avoid the possibility of unfair surprise, confusion of issues, and undue consumption of time. An issue is considered collateral if it would not be admissible other than to contradict the testimony. Evidence that a person has previously filed similar claims is generally inadmissible to show the invalidity of the present claim. At best, this evidence shows the plaintiff’s tendency toward litigation. Unless there is evidence that the previous claim was false, the probative value of such evidence is deemed outweighed by the risk of confusion of the issues. Because the prior suit would not be the subject of proof independent of impeachment, it is a collateral matter, and extrinsic evidence, such as the witness’s testimony, is inadmissible.

How well did you know this?
1
Not at all
2
3
4
5
Perfectly
7
Q

(*)

A mall leased one of its retail units to a clothing store for a period of five years. The lease agreement provided that the clothing store would pay to the mall, as additional rent, $1,000 a month in maintenance fees for the upkeep of the common areas in the mall. The agreement also permitted assignments and sublease.

For four years, the clothing store timely paid all rent and maintenance fees. At the end of the fourth year, the clothing store properly assigned the lease to a discount shoe outlet. At the time it assigned the lease, the clothing store owed $3,000 in maintenance fees for the last three months of its occupancy.

The shoe outlet paid its rent but did not pay any maintenance fees to the mall for the first six months. The shoe outlet then abandoned the property. The mall made reasonable efforts during the last six months of the term to relet the unit but was unable to do so. After applying the security deposit to satisfy the balance of the rent, the mall wishes to collect the unpaid maintenance fees for the last 15 months of the lease, totaling $15,000.

Who is liable for those fees and in what amount?

A The clothing store and the shoe outlet are jointly and severally liable for the $15,000 in fees.

B The clothing store is solely liable for $3,000 in fees, and the clothing store and the shoe outlet are jointly and severally liable for $12,000 in fees.

C The clothing store is solely liable for $3,000 in fees, the shoe outlet is solely liable for $6,000 in fees, and the clothing store and the shoe outlet are jointly and severally liable for $6,000 in fees.

D The clothing store is solely liable for $3,000 in fees, and the shoe outlet is solely liable for $12,000 in fees.

A

B

(assignment of leases.)
T2 was not in privity of estate when 3000 were owed)

The clothing store is liable for the total amount on privity of contract grounds, and the shoe outlet is liable for $12,000 on privity of estate grounds. When a leasehold interest is assigned, the assignor and the landlord are no longer in privity of estate; the assignee is now in privity of estate with the landlord. Hence, each is liable to the other on all covenants in the lease that “run with the land.” Here, the agreement to pay a maintenance fee for upkeep of the common areas of the mall is a covenant that runs with the land because it burdens the tenant and benefits the landlord with respect to their interests in the property (i.e., it “touches and concerns” the land). The shoe outlet is therefore liable for the maintenance fees for the 12-month term of its tenancy. However, because the shoe outlet was not in privity of estate prior to the assignment, it is not liable for the $3,000 in maintenance fees that the clothing store owed; thus, (A) is incorrect.

How well did you know this?
1
Not at all
2
3
4
5
Perfectly
8
Q

The federal government contracted with a number of communications utilities to install fiberoptic communication lines between major federal offices across the country. The utilities, which maintained ownership of the lines, contracted with the federal government to install the lines on a “cost plus fixed fee” basis, whereby all installation costs would be reimbursed by the government. One such line was installed in a state’s capital city, where the Department of the Interior maintained its western regional office. The state imposes a tax on the installation of all communication lines in the state, including fiberoptic cable lines. It seeks to impose the tax on the line running to the federal office.

Will the state be permitted to impose the tax?

A Yes, because the tax is indirect and nondiscriminatory.

B Yes, because the tax is a valid exercise of state power under the Tenth Amendment.

C No, because the tax burdens the activities of the federal government.

D No, because the activity taxed involves interstate commerce.

A

The state may impose a tax on the fiberoptic line.

A state tax levied directly against the property or operation of the federal government without the consent of Congress is invalid.
However, nondiscriminatory, indirect taxes on the federal government or its property are permissible if they do not unreasonably burden the federal government.

Because this tax is not levied directly against the government, but rather against the provider of a service that the government is obtaining, and is levied on all communications lines in the state, the tax is valid. The fact that the economic burden of the tax will ultimately be borne by the government under the “cost plus” contract does not invalidate the tax.

(C) is wrong because not every state tax that burdens the federal government is invalid. A nondiscriminatory tax on a service provided to the federal government by a private entity does not appear to be an unreasonable burden on the operation of the federal government.

How well did you know this?
1
Not at all
2
3
4
5
Perfectly
9
Q
A

State agency probationary employee does not have a substantial right under the procedural due process to get a hearing before being fired.
To have a property interest in continued government employment, there must be a statute, regulation, contract right, or clear policy that the employee can be dismissed only for cause. Absent such a right to employment, the employee is an at-will employee and may be terminated without a hearing.

The employee’s suit will be unsuccessful because he has no right to a hearing here because he has no life, liberty, or property interest at stake. The Due Process Clause requires a hearing only when a life, liberty, or property interest is at stake. The employee clearly is not at risk of losing his life or liberty, and the Supreme Court has made clear that neither is a property interest involved here.

To have a property interest in continued government employment, there must be a statute, regulation, contract right, or clear policy that the employee can be dismissed only for cause. Absent such a right to employment, the employee is an at-will employee and may be terminated without a hearing. Here, there was no law, contract, or policy giving the employee a right to a job absent cause for firing him. Therefore, no hearing was required. (A) is incorrect because bills of attainder involve criminal or otherwise punitive measures inflicted without judicial trial. Nothing here indicates that the employee is being punished; rather he is not being retained as an employee. (

How well did you know this?
1
Not at all
2
3
4
5
Perfectly
10
Q

<p>(*)
Police investigating a homicide had probable cause to believe that the defendant had committed it. They then learned from a reliable informant that, a short while ago, the defendant had gone to a friend’s house to obtain a false driver’s license from the friend, a convicted forger. Believing that the defendant might still be there, the police, without obtaining a warrant, went to the friend’s house. They entered the house and found the defendant hiding in the basement. He was arrested and given his Miranda warnings. At the police station, he confessed to the homicide.

At a preliminary hearing, the defendant’s attorney contends that the confession should be suppressed on Fourth Amendment grounds.

Is the court likely to agree?

A Yes, because the police did not have a search warrant to enter the friend’s house and there were no exigent circumstances.

B Yes, because the police did not have an arrest warrant for the defendant and there were no exigent circumstances.

C No, because a reliable informant told police that the defendant was in the friend’s house.

Correct
D No, because the police had probable cause to arrest the defendant.</p>

A

<p>Basically D could have argued that his 4th A were breached by D could not because it was not his house. D did not have stnaidng.

The court is not likely to agree to suppress the confession for two reasons: The defendant probably has no standing to raise a violation of the friend’s Fourth Amendment rights, and even if he did, the confession would not be excluded because it was not the fruit of the Fourth Amendment violation. Under the Fourth Amendment, the police generally can arrest, without a warrant, anyone that they have probable cause to believe has committed a felony. There are two exceptions, however, when a warrant is required: Absent exigent circumstances, the Fourth Amendment requires the police to have a warrant to arrest a person in his own home or to search the premises of a third person for an arrestee. Here, the police had probable cause to arrest the defendant and the arrest did not occur in the defendant’s home. Although the arrest did take place in the friend’s home and the police did not obtain a warrant to search the friend’s home, this will not help the defendant because the warrant requirement for a third person’s premises is intended to protect the third person’s expectation of privacy; while the search may have violated the friend’s Fourth Amendment rights, the friend is not being charged with an offense. The Supreme Court has held that a person can have evidence excluded on Fourth Amendment grounds only if that person’s Fourth Amendment rights were violated; a person has no standing to raise a violation of another’s Fourth Amendment rights. A person has standing to object to the search of a place only if the person has an ownership or possessory interest in the place searched or is an overnight guest in the place searched. Here, the police entered the friend’s home, and the facts indicate that the defendant was there only to obtain a forged license. Thus, the defendant probably has no standing to raise the Fourth Amendment violation. Furthermore, even if he had standing, his confession could still be used against him. While the exclusionary rule generally provides that evidence obtained or derived from exploitation of illegally obtained evidence must be excluded, the Supreme Court has held that where the police have probable cause to arrest a defendant and improperly arrest him in his home without a warrant, a confession made by the defendant at the police station is admissible because it is not the fruit of the unlawful arrest (because the police could have lawfully arrested the defendant the moment he stepped outside of the house). [New York v. Harris (1990)] Here, even if the arrest were unlawful because of the absence of a warrant, the police had probable cause to arrest the defendant and so could have waited until he left the friend’s house. Hence, the confession at the police station was not a fruit of the unlawful arrest and should not be suppressed.</p>

How well did you know this?
1
Not at all
2
3
4
5
Perfectly
11
Q

In a writing signed by both parties, a renowned architect agreed to design and supervise construction of a new house for a buyer. The architect’s fee was to be paid on completion of the house. When the design plans were about two-thirds complete, the architect assigned to a newly licensed architect “all of my rights and duties under my design and construction-supervision contract with the buyer.” The novice architect expressly promised the architect to carry out the work to the best of her ability. The buyer, on learning of the assignment, refused to allow the novice architect to proceed on the project and brought an action against the architect to compel him to resume and complete performance of the contract.

Is the buyer entitled to such relief?

A Yes, because the architect’s services under the contract are unique.

B Yes, because the architect has personally completed two-thirds of the design work.

C No, because the architect-buyer contract is one for personal services by the architect.

D No, because the architect effectively delegated his remaining duties under the architect-buyer contract to the novice architect.

A

Remember NO specific performance in services contract, because it would be involuntary servitude.

The buyer cannot compel the architect to resume performance. Contracts for personal services are not subject to specific performance notwithstanding the fact that damages might be inadequate or difficult to assess or the services to be performed are unique. The courts reason that specific performance of personal service contracts is tantamount to involuntary servitude and would present enforcement problems.

At most, the buyer would be able to obtain an injunction to prevent the architect from working on another project at the times the architect agreed to work for the buyer. Thus, (C) is correct and (A) is incorrect.

How well did you know this?
1
Not at all
2
3
4
5
Perfectly
12
Q

A foreign correspondent wished to purchase a parcel of land from a developer that was not yet on the market. Before he left the country, he gave his attorney $100,000 and his power of attorney.

He instructed the attorney that, should the land be put up for sale, she was authorized to: offer up to $100,000 for it, enter into a binding contract to purchase it on the correspondent’s behalf, and, if he did not return in time, close on the property. In early January, the developer put the land on the market. The attorney offered $75,000 for it, which the developer readily accepted.

On January 15, the attorney, on the correspondent’s behalf, entered into a written contract to purchase the land for $75,000. Closing was set for February 15. During this time, the attorney heard nothing from the correspondent. When he had not returned by the date of closing, the attorney attended the closing and tendered the $75,000. The developer tendered a deed made out to the correspondent as the grantee.

On February 20, news was received that the correspondent had been killed by a stray bullet on January 14. The correspondent’s will left his entire estate to his niece. The developer believes the conveyance to the correspondent is invalid, and brings a suit to quiet title to the land.

Who is the court most likely to find is the owner of the land?

A The niece, because the attorney held the deed on constructive trust for the correspondent’s estate.

B The niece, because of the operation of the doctrine of equitable conversion.

C The developer, because a deed to a nonexistent person is void and conveys no title.

D The developer, because the risk of loss is on the buyer.

A

Can’t make a contract with a dead person. If contractor dies after contract made then equitable conversion comes in otherwise not.

The developer owns the land because a deed to a nonexistent person is void and conveys no title. Because the correspondent was dead when the deed was delivered, the deed passed nothing and was a nullity. Note that the developer will be required to return the $75,000 to the correspondent’s estate to avoid unjust enrichment.

(A) is wrong because title never passed from the developer. Furthermore, even if it had, it would not have passed even bare legal title to the attorney because she was not a grantee and the developer did not intend to pass title to her.

(B) is wrong because the correspondent was killed before the contract was formed. Had he been alive at that time, the contract would have been valid and executory on his death.

If a buyer dies after the contract for sale was entered into but before it has been completed, his heirs or devisees can demand a conveyance of the land at the closing.

Because the correspondent was dead when the contract was entered into, however, the attorney’s agency was no longer valid and there was no contract.

How well did you know this?
1
Not at all
2
3
4
5
Perfectly
13
Q

Rule of priviledge of arrest in tort

A

For purposes of tort liability, a police officer is privileged to make an arrest without a warrant for a felony or for a breach of the peace committed in her presence, but not for a misdemeanor not involving a breach of the peace absent a statute providing otherwise. If the arrest is privileged, the officer may use only that degree of force necessary to effect the arrest, but never deadly force.

How well did you know this?
1
Not at all
2
3
4
5
Perfectly
14
Q

A client related to a lawyer the facts of a grievance against her former employer. After investigating the facts, the lawyer drafted and filed a complaint in federal district court, asserting an employment discrimination claim under federal employment statutes. When the lawyer signed and filed the complaint, she believed that the claim had legal merit based on having worked on a similar case a year before. The lawyer did not know that the relevant statutes had been amended in the interim and that the amendments definitively precluded her client’s claim. The defendant served its answer, asserting that the claim was without legal merit.

Did the lawyer violate Rule 11 of the Federal Rules of Civil Procedure when she signed and filed the complaint?

A No, because she believed in good faith that the legal contentions in the document were warranted.

B No, unless the lawyer persists in prosecuting the action once the amendment is brought to her attention.

C Yes, because she did not withdraw the complaint before the defendant served its answer.

D Yes, because the lawyer would have determined, had she conducted an appropriate inquiry before filing the action, that the legal contentions in the complaint were not warranted.

A

The lawyer violated Rule 11 when she signed and filed the complaint because, if she had conducted an appropriate inquiry, she would have determined that the legal contentions in the complaint were not warranted.

When a lawyer presents a pleading, written motion, or other paper to the court, she certifies that, to the best of her knowledge, information, and belief formed after an inquiry reasonable under the circumstances: (i) the pleading is not presented for an improper purpose; (ii) the legal contentions therein are warranted by existing law or a nonfrivolous argument for the modification of existing law or the establishment of a new law; (iii) the allegations and factual contentions either have, or upon further investigation or discovery are likely to have, evidentiary support; and (iv) denials of factual contentions are warranted on the evidence or, where specified, are reasonably based on lack of information and belief. Here, the legal contentions of the complaint are not warranted by existing law, and the lawyer would have discovered this if she had conducted a reasonable inquiry.

How well did you know this?
1
Not at all
2
3
4
5
Perfectly
15
Q

What is the consideration in a requirements contract?

A

THe fact that the person asking for the products can’t ask for anybody else.

How well did you know this?
1
Not at all
2
3
4
5
Perfectly
16
Q

farmer owned land in fee simple. He executed two deeds, the first conveying an undivided one-half interest in the land to a husband and a wife as joint tenants with right of survivorship, and the second conveying an undivided one-half interest in the land to the husband’s only child. The child was 13 years old at the time. The common law joint tenancy is unmodified by statute. The farmer handed the two deeds to the husband. The husband promptly and properly recorded the deed to himself and his wife and put the deed to his child in a safe-deposit box without recording it. No actual consideration was paid for the deeds. The same year, the husband, the wife, and the child were killed simultaneously in an airplane crash. They all died intestate.

The applicable statute in the jurisdiction provides that “when title to property or its devolution depends on priority of death and there is insufficient evidence that the persons have died otherwise than simultaneously, the property of each person shall be disposed of as if he had survived.”

An appropriate action was instituted by the heirs of the husband, the wife, and the child. The farmer, who is not an heir of any of the deceased, is a party to the action.

In whom should the court determine that title to the land is vested?

A Entirely in the farmer.

B One-half in the heirs of the husband and one-half in the heirs of the wife.

C One-half in the farmer, one-quarter in the heirs of the husband, and one-quarter in the heirs of the wife.

D One-half in the heirs of the child, one-quarter in the heirs of the husband, and one-quarter in the heirs of the wife.

A

D

The court should determine that title to the land is one-half in the heirs of the child, one-quarter in the heirs of the husband, and one-quarter in the heirs of the wife. The husband and the wife held the one-half interest in the land as joint tenants with right of survivorship. Thus, had one of them survived, he or she would own the entire one-half interest.

The operation of the simultaneous death statute in the jurisdiction, which disposes of property as if each survived, results in their property being distributed as though they were tenants in common; i.e., one-half of their interest passes through the husband’s estate as though he survived and one-half of their interest passes through the wife’s estate as though she survived.

The child always held her one-half interest as a tenant in common, so her one-half interest clearly passes to her heirs without any need to resort to the simultaneous death statute

How well did you know this?
1
Not at all
2
3
4
5
Perfectly
17
Q

A RAP question

A landowner was estranged from his son and three small grandchildren. The landowner owned a valuable piece of property that he wanted to pass on to his grandchildren, without his son’s involvement. The landowner conveyed the property to his own sister, “for life, remainder to all of my grandchildren who ever attain the age of 25.”

How can the grandchildren’s interest best be described?

A A contingent remainder.

B A vested remainder.

C An executory interest.

D Nothing.

A

<p>The grandchildren take nothing because the purported conveyance to them violates the Rule Against Perpetuities. Under the Rule Against Perpetuities, an interest in property is not valid unless it will vest, if at all, not later than 21 years after a life in being at the creation of the interest. The validity of interests under the Rule is determined at the time the interests are created, taking into account the facts then existing. The “lives in being plus 21 years” period begins to run, and the measuring lives used to show the validity of an interest must be in existence, at that time. The problem in this case is that there is an age contingency beyond age 21 in an open class. The perpetuities period begins to run on the date the landowner conveyed the property to his sister. After that date, the landowner’s son could have additional children, shortly after which the lives in being (the son and the three older grandchildren) might all die. The additional children’s interest would vest when they reach age 25, which is more than 21 years after lives in being.</p>

How well did you know this?
1
Not at all
2
3
4
5
Perfectly
18
Q

It is understood and agreed that the purchasers’ obligation to pay the $5,000 six months after the sale shall be voided if the current homeowners have not, within three months after the aforesaid sale, removed the existing pool in the rear of the house.”

Condition precedent in form or subsequent in form?

A

<p>A condition subsequent is one the occurrence of which cuts off an already existing duty of performance.

The form of the condition requiring removal of the pool is that of a condition subsequent because, under the language of the contract, failure to do so will cut off the buyers’ duty to pay the $5,000.

A condition precedent is one that must occur before an absolute duty of immediate performance arises in the other party.
The substance of the pool removal provision is that of a condition precedent because no duty to pay $5,000 arises until after the sellers have removed the pool.</p>

How well did you know this?
1
Not at all
2
3
4
5
Perfectly
19
Q

<p>D tour v frolick:

An electrician was employed by an electrical services company that had contracts with a number of large office and condominium buildings to provide emergency electrical services and repairs at any hour of the day or night. Hence, he was required to be “on call” 24 hours a day and to drive his company van, which had all of his tools, to his home each night. One afternoon, the electrician left the company’s office at 4 p.m. as usual. However, when he left the main highway, he did not turn left toward his home but instead turned right toward the supermarket a few blocks away to pick up some items for dinner. While leaving the supermarket parking lot, the electrician drove negligently and struck a pedestrian. The pedestrian suffered serious injuries and required several operations and a lengthy hospital stay. The pedestrian filed suit against the company for $100,000.

Is the pedestrian likely to recover from the company?

A Yes, because the electrician’s trip to the market was only a slight deviation from the direct route to his home.

B Yes, but only if the company knew that the electrician had proclivities to drive negligently.

C No, because turning in the opposite direction from his home constituted a “frolic” by the electrician.

D No, because an employer is not liable for the torts of an employee traveling to and from work.</p>

A

<p>The company probably will be vicariously liable to the pedestrian because the electrician’s deviation did not take him outside the scope of the employment relationship. Under the doctrine of respondeat superior, an employer will be vicariously liable for tortious acts committed by its employee if the tortious acts occur within the scope of the employment relationship.

What the scope of employment is in a particular case is a question of fact determined by factors such as the specific authorization by the employer, the employee’s motivation, and the normal routines of the employee.

Ordinarily, an employee heading home after work is no longer within the scope of employment. Here, however, the electrician was required to be “on call” 24 hours a day, and was required to drive the company van to his home so he would be ready to provide emergency service whenever a call would come in.

Most likely, then, the electrician was still within the scope of his employment when he was driving the van home. The next issue is whether his deviation from his route home took him outside the scope of his employment. Most courts today consider the foreseeability of the deviation to be the most important factor in determining whether the employee was still within the scope of employment or was on a “frolic” of his own. Thus, minor deviations in time and geographic area from the employer’s business are still within the scope of employment because they are foreseeable. Here, the electrician’s deviation of a few blocks from his normal route home to pick up some groceries was not a substantial enough departure from his employment purposes so as to be unforeseeable, and therefore the company can be held vicariously liable for the electrician’s negligence.</p>

How well did you know this?
1
Not at all
2
3
4
5
Perfectly
20
Q

<p>Interesting federal pre-emption

A state statute forbids the use of radar detecting devices while driving. A driver passing through the state while on vacation received a ticket for operating such a device. A valid federal administrative rule, adopted under a federal consumer product safety act, regulates the design of radar detection systems. The rule was issued to limit consumers’ exposure to electromagnetic fields. No other federal law applies.

Which of the following best states the effect of the federal rule on the state statute?

A The federal rule preempts the state statute, because the federal rule regulates the same subject matter—radar detectors.

B The federal rule preempts the state statute, because the federal rule does not contain affirmative authorization for continued state regulation.

C The federal rule does not preempt the state statute, because the state statute regulates local traffic rules, a field of exclusive state power.

D The federal rule does not preempt the state statute, because the purposes of the federal rule and the state statute are different.</p>

A

<p>The federal rule does not preempt the state statute. A valid federal statute or regulation may expressly or impliedly occupy the entire field regulated, so as to preclude even nonconflicting state or local regulation of the same general subject. Here, the federal rule relates to the subject of consumer product safety, while the state statute relates to traffic safety. Thus, even if the federal rule is deemed to occupy the entire field that it regulates, that field differs from the field to which the state statute relates. Consequently, there is no preemption. It follows that A incorrect.</p>

How well did you know this?
1
Not at all
2
3
4
5
Perfectly
21
Q

<p>The defendant is tried on a charge of driving while intoxicated. When the defendant was booked at the police station, a videotape was made that showed him unsteady, abusive, and speaking in a slurred manner.

If the prosecutor lays a foundation properly identifying the tape, should the court admit it in evidence and permit it to be shown to the jury?

A Yes, because it is a statement by an opposing party.

B Yes, because its value is not substantially outweighed by unfair prejudice.

C No, because the privilege against self-incrimination is applicable.

D No, because specific instances of conduct cannot be proved by extrinsic evidence.</p>

A

<p>This videotape, after being properly authenticated, would be considered to be real evidence going to show the intoxicated state of the defendant shortly after he was driving. It would be admitted as relevant because its value would not be substantially outweighed by undue prejudice. (A) is wrong because the videotape is not being offered to prove the truth of any statements that the defendant made; it is offered to prove only that he was intoxicated. Thus, it would not qualify as a statement by an opposing party. (C) is wrong because the defendant is not being asked to give any testimony. Rather, the matter in question is the introduction of real proof.</p>

How well did you know this?
1
Not at all
2
3
4
5
Perfectly
22
Q

<p>Question on concurrent conditions:

A brother and a sister made a written contract pursuant to which the brother promised to convey a specified apartment house to his sister in return for his sister’s promise (i) to convey a 100-acre farm to the brother and (ii) to pay the brother $1,000 in cash six months after the exchange of the apartment house and the farm.

Which of the following statements concerning the order of performances is LEAST accurate?

A The brother’s tendering of good title to the apartment house is a condition precedent to the sister’s duty to convey good title to the farm.

B The sister’s tendering of good title to the farm is a condition precedent to the brother’s duty to convey good title to the apartment house.

C The sister’s tendering of good title to the farm is a condition subsequent to the brother’s duty to convey good title to the apartment house.

D The brother’s tendering of good title to the apartment house and the sister’s tendering of good title to the farm are concurrent conditions.</p>

A

<p>The least correct statement refers to a condition subsequent because a condition subsequent is one the occurrence of which cuts off an already existing absolute duty of performance. The sister’s tendering of good title would not cut off the brother’s duty to perform. (A) and (B) are wrong because they are not inaccurate statements. A condition precedent is one that must occur before an absolute duty of immediate performance arises in the other party. When conditions are concurrent as they are here (as discussed below), it can be said that each condition is a condition precedent to the other. (D) is wrong because conditions concurrent are those that are capable of occurring together (which describes the brother and sister’s relationship regarding the exchange of the farm and the apartment house) and the parties are bound to perform at the same time. In effect, each is a condition precedent to the other.</p>

How well did you know this?
1
Not at all
2
3
4
5
Perfectly
23
Q

<p>Attempt vs. the actual crime

A statute in the jurisdiction makes it a crime to sell ammunition to a person under the age of 18. The courts have interpreted this statute as creating a strict liability offense that does not require knowledge of the age of the purchaser and as creating vicarious liability. A 16-year-old boy, who looked four or five years older, entered a store and asked a clerk for a box of .22-caliber shells. The store owner had instructed her employees not to sell ammunition to anyone under the age of 18 and to always ask for identification. The clerk asked the boy his age. The boy said he was 20. The clerk then placed a box of shells on the counter and asked, “Anything else?” The boy said that was all he wanted but then discovered he did not have enough money to pay for the shells, so the clerk put the box back onto the shelf.

If the owner of the store is charged with attempting to violate the statute, what would be her best argument?

A It was impossible for the sale to have occurred.

B She had strictly instructed her employees not to sell ammunition to minors.

C The boy lied about his age.

D The clerk did not have the mental state needed for the crime charged.</p>

A

<p>The store owner will not be convicted of an attempt to violate the statute if her employee did not have the requisite intent. Although the statute has been interpreted to create a strict liability crime, which does not require proof of criminal intent, an attempt of a strict liability crime requires proof that the defendant acted with the intent to bring about the proscribed result. Therefore, for the store owner to be charged vicariously with attempt, her employee must have acted with the requisite intent; he must have intended to sell the ammunition to a minor. If he did not so intend, the store owner will not be convicted of attempt</p>

How well did you know this?
1
Not at all
2
3
4
5
Perfectly
24
Q

<p>What happens when D lawyer sends a notice of deposition without subpoena to a non-party and the nonparty does not show up? Can P lawyer recover expenses of sending its lawyer to the deposition?</p>

A

<p>The plaintiff may recover reasonable expenses because the defendant’s attorney did not serve a subpoena on the witness. When a party who notices a deposition fails to serve a subpoena on a nonparty deponent who then does not appear, the opposing party may recover reasonable expenses for attending, including attorney’s fees.</p>

How well did you know this?
1
Not at all
2
3
4
5
Perfectly
25
Q

<p>Can a landlord burn his own building that is currently being rented?</p>

A

<p>NO The requirement that the building be “of another” pertains to possession rather than ownership. Thus, a landlord could be guilty of arson for burning down his own building if his tenants were in possession of it rather than him;</p>

How well did you know this?
1
Not at all
2
3
4
5
Perfectly
26
Q

<p>A woman and her boyfriend worked together as pickpockets. The woman approached a man from the front to distract him, holding a small camera and asking him to take a picture, while her boyfriend came up from behind with a knife to slice open the back pocket of the man’s pants and remove his wallet. The man was drunk and believed that the woman had a gun and was trying to rob him, but he was unaware of the boyfriend behind him. The man reached into his back pocket to hand over his wallet and was cut by the boyfriend’s knife as it was slicing through his pocket. The wallet dropped to the ground as the man clutched his hand. The boyfriend picked it up, and he and the woman fled while the man knelt on the ground in pain. The woman was apprehended shortly thereafter and charged with robbery.

Should the woman be found guilty?

A Yes, because her boyfriend obtained the property by means of force.

B Yes, because the man believed that the woman would shoot him if he did not give up his wallet.

C No, because neither the woman nor her boyfriend intended to use force against the man to obtain the property.

D No, because the man’s belief that the woman was robbing him was unreasonable.</p>

A

<p>The woman should be found guilty of robbery because her accomplice obtained the wallet by means of force. Robbery consists of (i) a taking (ii) of personal property of another (iii) from the other’s person or presence (iv) by force or intimidation (v) with the intent to permanently deprive him of it.

Thus, robbery is basically an aggravated form of larceny in which the taking is accomplished by force or threats of force. The force must be used either to gain possession of the property or to retain possession immediately after such possession has been accomplished, but the defendant need not have intended to use force to complete the crime; the only intent required is the intent to permanently deprive the victim of his property. Here, the woman and her boyfriend had such intent, and they were able to carry out that intent in part because the boyfriend slashed the man’s hand with the knife, incapacitating him.

The fact that the boyfriend did not intend to injure the man is irrelevant; hence, (C) is wrong. (B) is wrong because the man’s erroneous belief that he was being threatened does not establish the element of threat or intimidation. The woman’s conduct was merely an attempt to distract the man and did not constitute a threat or intimidation; the fact that the man’s intoxication caused him to believe otherwise does not change that result. (D) is wrong because the unreasonableness of the man’s belief does not change the fact that the woman is liable as an accomplice to the robbery by her boyfriend, because robbery (the use of force) was a foreseeable consequence of the pickpocketing.</p>

How well did you know this?
1
Not at all
2
3
4
5
Perfectly
27
Q

<p>Implied in fact contracts

A professor who was an expert on American history conducted full-day tours through the historic sites of Philadelphia. The professor's fee for his services was $105, which did not include the entrance fees for several of the historical sites. A law student took a day off school and "hung around" the Liberty Bell monument, where the professor's tour started. That day the professor was conducting 27 persons on the tour. Most of the participants had paid in advance, but the professor was holding up a sign with information about the tour and handing out brochures, one of which the law student took. The professor accepted a few additional participants who signed up on the spot, but the law student was not among them. All day long, the law student hung around at the fringe of this group, paying the entrance fees separately but following the group through the different historical sites. However, he always positioned himself close enough to the professor's group so that he could hear virtually every word of the professor's lecture, although the law student did not ask the professor any questions. The law student signed his name and address on the register at Independence Hall. The professor noted this and took down the information. Two days after the tour concluded, the law student received a bill from the professor in the amount of $105.

How much will the law student most likely be required to pay the professor?

A $105, because that is the contract price for the tour.

B $105, because the amount of the contract was less than $500, making the Statute of Frauds inapplicable.

C $105, if that is a reasonable fee for the lectures based on the professor's expertise.

D Nothing, because the historical sites were open to the public and the law student paid his own way.</p>

A

<p>The law student will probably be required to pay the professor $105 under an implied-in-fact contract. An implied-in-fact contract is a contract formed by manifestations of assent other than oral or written language, i.e., by conduct. Even if there is no subjective "meeting of the minds," the parties will be bound if their conduct objectively appears to manifest a contractual intent. Where an offeree silently takes the benefit of offered services with reasonable opportunity to reject them and reason to know that they were offered with the expectation of compensation, the offeree's inaction may constitute an acceptance. [Restatement (Second) of Contracts §69(1)(a)] Here, the student's silence in the face of the professor's offer and his conduct in staying within earshot of the group is a sufficient objective manifestation of contractual intent for the court to find an implied-in-fact contract. Hence, a court will probably allow the professor to recover the contract price. (B) is wrong because it states the wrong rationale. The Statute of Frauds would not be applicable even if the cost of the tour were over $500; the $500 provision of the Statute of Frauds is applicable only to the sale of goods. (C) is wrong because it states a restitutionary remedy available in a quasi-contract action. A quasi-contract action for restitution is a legal remedy to prevent unjust enrichment where an enforceable contract is not present, and allows the claimant to recover the reasonable value of the benefits that he rendered to the other party. While the professor probably could pursue a quasi-contract action for restitution because he rendered services with a reasonable expectation of being compensated, he is not limited to that remedy because he can establish an implied-in-fact contract. Hence, he can recover the contract price for the tour without having to establish that it was a reasonable fee for the lectures.</p>

How well did you know this?
1
Not at all
2
3
4
5
Perfectly
28
Q

<p>Fifteen years ago, a homeowner executed his will, devising his home “to my surviving widow for life, remainder to such of my children as shall live to attain the age of 30 years; but if any child dies under the age of 30 years survived by a child or children, such child or children shall take and receive the share which his, her, or their parent would have received had such parent lived to attain the age of 30 years.” At the date of writing his will, the homeowner was married to an actress, and they had two adult daughters. The actress died 10 years ago, and the homeowner married a dancer two years later. At his death last year, the homeowner was survived by the dancer and three children, the two daughters from his marriage to the actress, and a son. The son, who is six years old, was the homeowner’s child by the dancer. The jurisdiction recognizes the common law Rule Against Perpetuities unmodified by statute.

What is the result of the application of the Rule?

A The remainder to the children and to the grandchildren is void, because the homeowner could have subsequently married a person who was unborn at the time he executed his will.

B The remainder to the children is valid, but the substitutionary gift to the grandchildren is void, because the homeowner could have subsequently married a person who was unborn at the time he executed his will.

C The gift in remainder to the daughters or their children is valid, but the gift to the son or his children is void.

D The remainder to the children and the substitutionary gift to the grandchildren are valid.</p>

A

<p>The gifts are valid under the Rule. The homeowner’s will created a life estate in the dancer, contingent remainders in the class consisting of the homeowner’s children (contingent upon their attaining age 30), and contingent remainders in the class consisting of any children of the homeowner’s children (contingent on their surviving their parent, and the parent dying before attaining age 30). There are two keys to understanding the question. The first is that a will speaks at death, no matter when it was executed. Here, the homeowner’s will became an effective conveyance only when he died last year. The second key is that the grandchildren (i.e., the children of the daughters or the son) do not themselves have to survive to any particular age to take their gifts. The wording of the question is somewhat confusing on this point, but it is clear when read carefully. Because there are two future interests in the question, each must be analyzed separately under the Rule Against Perpetuities. The gift to the homeowner’s children is a class gift, and the Rule makes class gifts entirely void unless it is certain that the gift will vest or fail as to all members of the class within the perpetuities period. However, it is clear that this will be true here. The three children (the daughters and the son) are all alive when the will speaks. Hence, they are all lives in being. (If the dancer had been pregnant when the homeowner died, that child, when born, would also have been considered a “life in being” as of the homeowner’s death.) The gift is certain to vest as to each of the homeowner’s children when each reaches age 30, which is obviously within each child’s lifetime. Likewise, if one of the children dies before age 30, his or her interest will fail; again, that is certain to happen within his or her lifetime. Because this is so, the class gift to the children of the homeowner is certain to vest or fail as to each member within “lives in being.” The gift is therefore valid under the Rule. It is not even necessary to add the 21-year period as permitted by the Rule. As to the class gift to the grandchildren of the homeowner, a similar analysis follows. If any grandchild’s interest ever becomes vested, it will do so immediately on the death of that grandchild’s parent (one of the daughters or the son) prior to reaching age 30. Because those three persons are “lives in being” at the homeowner’s death, the grandchildren’s interests are certain to vest or fail in every case at the end of a life in being. Again, it is not necessary to add the 21-year period as permitted by the Rule.

(A) is wrong because the time of execution of the will is irrelevant; it is the date of the testator’s death that commences the running of the perpetuities period.

(B) is wrong for the same reason.

(C) is wrong for the reasons discussed above.</p>

How well did you know this?
1
Not at all
2
3
4
5
Perfectly
29
Q

<p>An electrical engineer designed an electronic game. The engineer entered into a licensing agreement with a manufacturer under which it agreed to manufacture the game according to the engineer’s specifications and to market it and pay a royalty to the engineer. A girl whose parents had purchased the game for her was injured while playing with the game. The girl recovered a judgment against the manufacturer on the basis of a finding that the game was defective because of the engineer’s improper design. Evidence was also presented that the manufacturer could have discovered the defect by reasonable inspection.

In a claim for indemnity against the engineer, will the manufacturer prevail?

A Yes, because as between the engineer and the manufacturer, the engineer was responsible for the design of the game.

B Yes, because the manufacturer and the engineer were joint tortfeasors.

C No, because the manufacturer was strictly liable to the girl.

D No, because the manufacturer, by a reasonable inspection, could have discovered the defect in the design of the game.</p>

A

<p>The manufacturer will prevail. Generally, a joint tortfeasor may recover indemnification from another joint tortfeasor where there is a considerable difference in the degree of fault. Here, the engineer, the person whose improper design actually caused the girl’s injuries, is a “more wrongful” tortfeasor than the manufacturer. Thus, the manufacturer should prevail in its claim against the engineer, which result is reflected in (A).

(B) is incorrect because indemnity is not available simply because the manufacturer and the engineer are joint tortfeasors.

(C) is incorrect because the manufacturer’s liability to the girl does not preclude it from obtaining indemnity from the engineer.

(D) is incorrect because, even though the manufacturer was negligent in failing to discover the defect, it may still be entitled to indemnity from the person who negligently designed the game as the “more wrongful” tortfeasor.</p>

How well did you know this?
1
Not at all
2
3
4
5
Perfectly
30
Q

<p>A defendant was charged with murder. His principal defense was that he had killed in hot blood and should be guilty only of manslaughter. The judge instructed the jury that the state must prove guilt beyond a reasonable doubt, that the killing was presumed to be murder, and that the charge could be reduced to manslaughter, and the defendant accordingly found guilty of this lesser offense, if the defendant showed by a fair preponderance of the evidence that the killing was committed in the heat of passion on sudden provocation. The defendant was convicted of murder. On appeal, he seeks a new trial and claims error in the judge’s instructions to the jury.

Will the defendant’s conviction be affirmed?

A Yes, because the judge carefully advised the jury of the state’s obligation to prove guilt beyond a reasonable doubt.

B Yes, because the defendant’s burden to show hot blood was not one of ultimate persuasion but only one of producing evidence to rebut a legitimate presumption.

C No, because the instruction put a burden on the defendant that denied him due process of law.

D No, because presumptions have a highly prejudicial effect and thus cannot be used on behalf of the state in a criminal case.</p>

A

<p>The conviction should be reversed. The Due Process Clause has been interpreted as requiring the prosecution to prove each element of the crime charged beyond a reasonable doubt. The “malice aforethought” element of murder has traditionally been defined as encompassing the absence of provocation engendering a passion. Putting the burden of persuasion as to the existence of provocation and passion on the defendant relieves the prosecution of its burden as to their absence.

Therefore, (C) is the best answer and (A) is incorrect on the facts.

(B) is incorrect because the presumption of “malice aforethought” is not a legitimate presumption. (D) is incorrect as a matter of law. Presumptions are permitted as long as they are not mandatory for the jury.</p>

How well did you know this?
1
Not at all
2
3
4
5
Perfectly
31
Q

<p>What do you get for partial performance?</p>

A

<p>In such cases where the builder breaches after partially performing, the owner of the land is entitled to the cost of completion plus reasonable compensation for any delay in performance. Courts generally allow the builder to offset or recover for work performed to date to avoid the unjust enrichment of the owner. Hence, the unpaid installments should be deducted.</p>

How well did you know this?
1
Not at all
2
3
4
5
Perfectly
32
Q

<p>Example of issue preclusion:

A motorcyclist, a car driver, and a truck driver were involved in a three-vehicle accident in a busy intersection. The motorcyclist filed a negligence action against the car driver for personal injuries suffered in the accident. In the car driver's pleadings, she denied that she was negligent and raised the motorcyclist's contributory negligence as a defense. The jury returned a general verdict in favor of the car driver. The motorcyclist then filed a negligence action against the truck driver. At the appropriate time, the truck driver filed a motion to dismiss for failure to state a claim, asserting that the motorcyclist was precluded from re-litigating his contributory negligence.

Is the truck driver likely to be successful?

A Yes, because there was a final judgment on the merits in the first case.

B Yes, because the jury in the first case determined that the motorcyclist was contributorily negligent.

C No, because it is not clear whether the jury in the first case found that the motorcyclist was negligent.

D No, because only someone who was a party in the first action can assert issue preclusion in the second.</p>

A

<p>The truck driver is unlikely to be successful because it is unclear whether the jury in the first case found that the motorcyclist was contributorily negligent. For issue preclusion to apply to an issue, the issue must actually have been litigated and determined in the previous case. Here, the issue of the motorcyclist’s negligence was not determined in the first case. The jury could have found that the motorcyclist was contributorily negligent, or it simply could have found that the motorcyclist did not prove that the car driver was negligent. Because this issue was not decided, the truck driver cannot assert issue preclusion (collateral estoppel) against the motorcyclist. (B) is therefore incorrect.</p>

How well did you know this?
1
Not at all
2
3
4
5
Perfectly
33
Q

<p>Can a defeasible fee simple owner take minerals out?</p>

A

<p>Yes.The owner of a defeasible fee has the same right to possession and privileges of use as the owner of a fee simple absolute.</p>

How well did you know this?
1
Not at all
2
3
4
5
Perfectly
34
Q

<p>A 10-lot subdivision was approved by the proper governmental authority. The authority’s action was pursuant to a map filed by the developer, which included an undesignated parcel in addition to the 10 numbered lots. The shape of the undesignated parcel is different and somewhat larger than any one of the numbered lots. Subdivision building restrictions were imposed on “all the lots shown on said map.” The developer contracts to sell the undesignated parcel, described by metes and bounds, to an investor.

Is title to the parcel marketable?

A Yes, because the undesignated parcel is not a lot to which the subdivision building restrictions apply.

B Yes, because the undesignated parcel is not part of the subdivision.

C No, because the undesignated parcel has never been approved by the proper governmental authority.

D No, because the map leaves it uncertain whether the undesignated parcel is subject to the building restrictions.</p>

A

<p>Title to the undesignated parcel is unmarketable because the map leaves it uncertain whether the parcel is subject to the building restrictions. Marketable title is one that is free from reasonable doubt in fact or in law. Here, there is confusion because the building restrictions apply to all the lots shown on the map, but the parcel at issue is not one of the 10 numbered lots. Thus, it is unclear whether the parcel is subject to restrictions that will reduce the uses of the lot or its market value.

(A) is wrong because it is not clear that the undesignated parcel is not subject to the subdivision restrictions. It was included on the map and the restrictions apply to “all lots shown.”

(B) is wrong because even though the undesignated parcel is not one of the 10 lots, it may be bound by the restrictions. Because this is unclear, title is not marketable.

(C) is wrong because there could be marketable title without government approval.</p>

How well did you know this?
1
Not at all
2
3
4
5
Perfectly
35
Q

<p>(*)
On March 15, a vineyardist entered into a written agreement with a winery that provided that the vineyardist would sell 1,600 tons of tokay grapes to the winery for $750 per ton, delivery to be no later than November 1 of the same year. By November 1, the vineyardist had delivered only 700 tons of grapes and had informed the winery by fax that she had used the remainder of her crop in the production of wine for her own shop. The winery purchased an additional 900 tons of tokay grapes from other growers at the then-prevailing market price of $800 per ton. The vineyardist has submitted an invoice to the marketing department of the winery for $525,000.

Ignoring incidental costs of cover, how much should the winery pay the vineyardist?

A $525,000, since by accepting delivery of the 700 tons of grapes, the winery waived an objection to the vineyardist’s breach.

B The market value of her 700 tons of grapes as of November 1, less the cost of cover for the remaining 900 tons.

C $480,000, which represents the contract price for the grapes she delivered less the cost of cover for the remaining 900 tons.

D Nothing, since she will be unable to enforce any claim for payment in court.</p>

A

<p>The vineyardist is entitled to the contract price for the grapes delivered and accepted, but the winery is entitled to cover—to purchase grapes at the market price prevailing at the time of performance and to deduct any increase over the contract price. (A) is wrong because the winery is entitled to cover and does not “waive” the breach by accepting a part performance. (B) is wrong because the price for the grapes delivered is the contract price, not the prevailing market price. (D) is wrong because the vineyardist is entitled to payment for the grapes she delivered and would be able to enforce her claim through litigation, if necessary.</p>

36
Q

<p>A state statute permits the state to seize and dispose of real property that was used to commit or facilitate the commission of a felony drug offense. After a drug dealer’s arrest for selling cocaine out of his home, a felony, the state instituted an action of forfeiture against the drug dealer’s house and property. After notice to the drug dealer and a hearing, a judge granted the order and the state seized the property. Six months later, after the time for any appeals had expired, the property was sold at a public auction to a third party. It was only when the third party brought an action to quiet title that a bank holding a properly recorded mortgage on the drug dealer’s property learned of the forfeiture. Because the bank’s mortgage payments were automatically deducted from an account the drug dealer had under a different name, no one at the bank was aware that the property had been seized. The only notice provided to parties other than the drug dealer was a public notice published for three weeks in a general circulation newspaper. The bank defends the quiet title action on the ground that it did not receive the notice required under the United States Constitution to protect its interest in the property.

If the court rules that the bank’s rights under the Due Process Clause of the Fourteenth Amendment were violated by the state’s seizure of the property, what is the most likely reason?

A In any judicial proceeding affecting rights to real property, a claimant is required to provide notice and an evidentiary hearing to all parties with a legal interest in the property before taking actions affecting their rights.

B The government itself was the party that seized the property, rather than a private party using governmental processes.

C The notice was not adequate under the circumstances to apprise a party with a properly recorded legal interest in the property.

D The jurisdiction treats the mortgagee as having title to the property rather than merely a lien.</p>

A

<p>The basis for finding that the bank’s due process rights have been violated is that it should have received notice through personal service or by mail. When the government seeks to use a judicial or administrative process to take or terminate property interests, it must give notice to those persons whose property interests may be taken by that process. The form of notice must be reasonably designed to insure that those persons will in fact be notified of the proceedings. Here, the bank had recorded its mortgage and presumably could have been notified by mail that the property was being seized by the government. Being deprived of the opportunity to protect its interest in the property violated the bank’s due process rights under the three-part test of Mathews v. Eldridge. Mathews lists three criteria that the courts should weigh in determining what constitutes fair process: (i) the importance of the individual interest involved, (ii) the value of specific procedural safeguards to that interest, and (iii) the governmental interest in fiscal and administrative efficiency. Here, the bank has an important property right that is being terminated, the procedure of publishing a general notice was not sufficient to safeguard its interests, and the government interest in efficiency would not have been overburdened by requiring notice by mail to parties with a recorded interest in the property.

(A) is incorrect because it is too broad. Under Mathews, the government is not required to provide personal notice to all parties if it is not feasible, nor is it required to provide a preseizure hearing if exigent circumstances make it impracticable.

(B) is incorrect because even when a private party is seeking to use a judicial or administrative process, state action is involved and the Due Process Clause must be satisfied; hence, notice to a record mortgage holder would be required even if a private party were seeking to seize the property through judicial means.

(D) is incorrect because the state’s characterization of the mortgagee’s interest is not critical; the mortgagee has legal rights to the property that are protected by the Due Process Clause, regardless of how the rights are characterized.</p>

37
Q

<p>A lawyer was appointed as an administrative judge to review claims against the federal government made by Native Americans under a congressional statute. For 20 years, the lawyer heard, reviewed, and arbitrated disputed claims made against the government by various Indian tribes and their citizens. When the lawyer found a claim to be valid, he would make a recommendation to the Bureau of Indian Claims that the claim be paid. If the lawyer found the claims to be without merit, or if the Bureau decided against his recommendation, the claimant would have the right to bring suit in a federal court. Last year, a presidential commission recommended the abolition of the Bureau of Indian Claims as a cost-cutting measure. Congress acted on this recommendation and repealed the statute. The lawyer was offered a position as an attorney in the Department of Transportation, but he turned it down and brought suit against the government.

What is the likely result of this suit?

A The lawyer prevails, because it violates the doctrine of separation of powers for the executive branch to interfere with a congressional act by recommending its repeal.

B The lawyer prevails, because it violates the Constitution to terminate the tenure of a federal judge during good behavior.

C The government prevails, because it established the lawyer’s position and it can terminate it at will.

D The government prevails, because the lawyer had no judicial discretion or powers in his position with the Bureau.</p>

A

<p>The government will most likely prevail because the lawyer had no judicial discretion or powers in his position. Under Article III of the Constitution, a federal judge is protected from termination of tenure during good behavior. This necessarily requires that a person who seeks protection under this provision be able to show that he is a federal “judge.” From the facts, the lawyer was clearly no more than an administrative hearing officer, without discretion or power. Thus, he would not be a judge within the meaning of this article, and its provisions would not apply to him. Therefore (B) is wrong.

(A) is wrong because anybody can recommend that Congress enact or repeal a statute. Just because an executive branch’s commission does so does not mean that there is a violation of the separation of powers doctrine.

(C) is factually incorrect and does not explain the proper reason for the result.</p>

38
Q

<p>what are the elements of misrepresentation (tort)? and do you need a pecuniary loss?</p>

A

<p>YES you do (elements you should know by know or look them up moron)</p>

39
Q

<p>What happens if lawyer accidentally sends priviledge information?</p>

A

<p>If a trial party inadvertently discloses privileged material to an opposing party, it may still invoke a claim of privilege by notifying the opposing party of the disclosure and the basis for the claim of privilege. Once notified, the opposing party must promptly return, sequester, or destroy the specified information and take reasonable steps to retrieve the material if it disclosed it to others. The opposing party also may not use or disclose the privileged material until the claim is resolved.</p>

40
Q

<p>Does "heat of passion" apply here?

After an altercation on the practice field between a freshman and a senior on a college football team, the senior came up behind the freshman in the locker room and shoved him in the back. When the freshman turned around, the senior punched him in the face. Before the freshman could retaliate, the two were quickly separated by other players. The senior taunted him, “The next time I see you I won’t go so easy on you.” That night the freshman was at a bar frequented by the football players and became enraged when some of them teased him about the altercation. He then saw the senior enter the room but kept his back to him. Suddenly he felt someone shove him in the back. Pulling out his pocketknife, he whirled and stabbed the person behind him, believing it to be the senior. In fact, it was an intoxicated patron who had stumbled and fallen into the freshman. The knife severed a major artery, and the patron died on the way to the hospital.

The freshman is charged with murder for the patron’s death. At trial, the freshman testified that he honestly believed that the senior was going to kill him the next time he saw him. On cross-examination, he admitted that such a belief was unreasonable. The freshman’s attorney requests the judge to instruct the jury on voluntary manslaughter, both on an “imperfect self-defense” theory and on a “heat of passion” basis.

How should the judge respond?</p>

A

<p>The judge should give both manslaughter instructions and allow the jury to consider both theories of manslaughter in determining whether the intentional killing should be reduced to voluntary manslaughter. In a “heat of passion” killing, provocation will reduce a killing to voluntary manslaughter if four requirements are met: (i) the provocation was a type that would arouse sudden and intense passion that would cause a reasonable person to lose self-control, (ii) the defendant in fact was provoked, (iii) there was not sufficient time between the provocation and the killing for the passion of a reasonable person to cool, and (iv) the defendant in fact did not cool off between the provocation and the killing. Although some provocations were defined as inadequate as a matter of law at common law, modern courts are more likely to submit to the jury the question of what constituted adequate provocation. Similarly, whether there has been a sufficient time for a reasonable person to cool off is a factual question that depends on the nature of the provocation and the attendant circumstances. Here, the freshman’s belief that he was being shoved again by the senior and set up for a more severe beating may have rekindled his rage at the earlier punch and taunting by the senior. The jury should be allowed to consider all of the circumstances, including the earlier altercation, to decide whether there was a sufficient provocation or a sufficient time for a reasonable person to cool off. Hence, the judge should agree to give the “heat of passion” manslaughter instruction.

Some states recognize, as this state apparently does, an “imperfect self-defense” doctrine under which a murder may be reduced to manslaughter even though the defendant unreasonably but honestly believed in the necessity of responding with deadly force. Such a defense appears to also have been raised by the facts.</p>

41
Q

<p>Larceny or continuing trespass?

A woman was in the process of moving out of the apartment that she had shared with a roommate. She collected numerous items of hers from her roommate’s room that the roommate had borrowed. As she was leaving the apartment, she grabbed what she believed to be her laptop computer, which her roommate had often borrowed. Because it was an older, slower machine, she planned to trade it in for a different model at a computer resale store. She noticed that the laptop was much lighter than usual, but she reasonably attributed this to her diligence in following a weight-lifting regimen at her gym. When she arrived at the computer store, she discovered that she had taken a brand new, state-of-the-art laptop that her roommate had recently purchased. She then kept the laptop rather than buying a new one.

Is the woman guilty of common law larceny?

A No, because she mistakenly believed that the computer she had picked up was hers.

B No, because her mistake as to whose computer she had picked up was reasonable.

C Yes, because she intended to keep the computer when she took it.

D Yes, because she decided to keep the computer when she discovered the mistake.</p>

A

<p>The woman is not guilty of common law larceny of the computer because her mistake prevented her from having the requisite mens rea for larceny. Larceny requires the intent to permanently deprive another of her interest in the property taken. The woman did not have such intent, given that she believed that the computer was her own and that her roommate had no possessory interest in it. Therefore, she did not have the intent required for larceny. (B) is wrong because the woman’s mistake need not have been reasonable. When mistake is offered to negate the existence of general intent or malice, it must be a reasonable mistake. However, any mistake of fact, reasonable or unreasonable, is a defense to a specific intent crime, and larceny is a specific intent crime. (C) is wrong because, as stated above, she did not have the intent to deprive her roommate of her roommate’s computer; her mistake negates such intent.

(D) is wrong because the “continuing trespass” doctrine is inapplicable. Although larceny generally requires the intent to deprive another person of her interest in the property at the moment of taking, the continuing trespass doctrine provides that if a defendant takes property with a wrongful state of mind, but without the intent to steal, and then he later forms the intent to steal it, the trespass involved in the initial wrongful taking is regarded as “continuing” and the defendant is guilty of larceny. However, this doctrine has no application if the defendant’s initial taking of the property, although trespassory, was not motivated by a wrongful state of mind. Here, the woman took her roommate’s computer as a result of an innocent mistake. Even though she decided to keep the computer, she will not be guilty of larceny because her initial taking was done with an innocent state of mind.</p>

42
Q

<p>A small village had a population of 400, and 90% of the inhabitants belonged to the same church. The village council consisted of five members, all of whom were members of that church. The council unanimously appropriated $350 to pay a landscaper to mow their church’s lawn for a year. An atheist in the village, who lived in a tiny house and paid only $200 per year in village and county property taxes, was incensed that any of it should go to support the church. The atheist filed suit in federal district court to strike down the council’s appropriation to mow the church lawn.

Should the court entertain the atheist’s suit?

A No, because the atheist paid so little in taxes that his interest in the matter, if any, is too minimal.

B No, because taxpayers lack standing to sue over appropriations by duly constituted legislative bodies.

C Yes, because a taxpayer may sue under the authority of the First Amendment’s Establishment Clause if a fund into which he has paid is being used for religious purposes.

D Yes, because taxpayers have standing to sue when questions involving constitutional rights are at issue.</p>

A

<p>Because the appropriation by the village council implicates the Establishment Clause and a portion of the atheist’s taxes are paying for the appropriation, the court should entertain his suit. To raise a constitutional challenge to a government action, the person who is challenging it must have standing to raise the constitutional issue. To have standing, a person must be able to assert that he is injured by a government program. This injury must be more than the merely theoretical injury that all persons suffer by seeing their government engage in unconstitutional actions. In cases where a taxpayer is claiming injury from the use of his taxes for an unconstitutional appropriation or expenditure of public funds, the Supreme Court has required the taxpayer to show that the challenged measure (i) was enacted under the governmental body’s power to tax and spend (rather than as an incidental expenditure in the administration of an essentially regulatory statute), and (ii) exceeds some specific limitation on that power. The only specific limitation that the Court has recognized when an appropriation measure has been challenged is the First Amendment’s Establishment Clause. [See Flast v. Cohen (1968)] Here, the atheist is challenging a specific appropriation of money by the village council that has been earmarked to benefit the property of the local church. The Establishment Clause, which is applicable to the states through the Fourteenth Amendment, requires that government programs (i) must have a secular purpose, (ii) must have a primary effect that neither advances nor inhibits religion, and (iii) must not produce excessive government entanglement with religion. Because the council’s appropriation may violate these requirements, the atheist has standing as a taxpayer to sue the council.</p>

43
Q

<p>A carload of gang members, armed with automatic rifles, sped into the rival gang’s neighborhood one night looking for a store that its rival gang used as a hangout. The gang members knew that the store closed at 6 p.m. and that the rival gang usually went elsewhere after nightfall. As the gang members drove by, they sprayed the building with bullets. One of the bullets struck and killed a six-year-old girl who was asleep in an apartment located on the second floor above the corner store. A few days later, the police arrested the defendant, who admitted to being a member of the gang and to having been in the car when the gang members shot up the store. The defendant was placed on trial for the girl’s murder.

If the defendant takes the stand in his own defense, and the jury believes the defendant’s testimony, which of the following assertions by the defendant would be his best defense to the murder charge?

A “I was the driver of the car and did not actually shoot into the building.”

B “I took a lot of drugs that night, and I was so high that I don’t even remember the incident; I did not intend to kill somebody.”

C “Another member of my gang pointed a gun at me. I was really scared that if I didn’t shoot into the building, I would be seriously injured or killed myself.”

D “I believed that the building was abandoned and had no idea that there would be people inside it.”</p>

A

<p>If the defendant believed that the building was abandoned, he probably will not be found to have had the requisite mens rea for murder. Murder is the unlawful killing of another human being with malice aforethought. Malice aforethought exists when the defendant has (i) intent to kill, (ii) intent to inflict great bodily injury, (iii) reckless indifference to an unjustifiably high risk to human life (“abandoned and malignant heart”), or (iv) intent to commit a felony (under the felony murder doctrine). Here, the jury could find that spraying a building with gunfire from a submachine gun demonstrates a reckless indifference to an unjustifiably high risk to human life even though the defendant was not intending to shoot anyone. However, if the jury accepts his assertion that he believed that the building was abandoned and had no idea that there would be any people inside it, the jury will probably find that he did not have a sufficient awareness of unjustifiably high risk to human life to be liable for murder.

(B) is incorrect because the defendant’s voluntary intoxication would not be a defense to a murder charge that was based on reckless indifference to an unjustifiably high risk to human life. Voluntary intoxication caused by alcohol or drugs is a defense to a crime that requires purpose (intent) or knowledge as long as the intoxication prevents the defendant from formulating the purpose or obtaining the knowledge. It is no defense to crimes involving recklessness or negligence, however. Even though recklessness requires a conscious disregard of the risk, and the defendant’s intoxication may make him unaware of the risk, courts hold him liable for recklessness offenses because his initial act of becoming voluntarily intoxicated is deemed reckless under the circumstances. Thus, for murder based on a reckless indifference to an unjustifiably high risk to human life, voluntary intoxication would not be a defense.</p>

44
Q

<p>An entrepreneur purchased a piece of undeveloped land with plans to build a luxury spa. He financed the purchase with a loan from a bank, secured by a mortgage on the land. The land contained a mineral hot spring. The entrepreneur hired builders to harness the spring’s water into a soaking pool. The spa was not a success and the entrepreneur ran out of money. He entered into a contract that purported to transfer his inventory and all his interests in the soaking pool to a buyer by a document that was sufficient as a bill of sale to transfer personal property but was insufficient as a deed to transfer real property. The bank soon after foreclosed on its mortgage and the land was sold at auction to a bidder, who took title in fee simple.

Who has title to the soaking pool?

A The bidder, as fee simple owner of the land.

B The buyer, as purchaser of the soaking pool under the bill of sale.

C The person who owns the water rights as an incident thereto.

D The entrepreneur, as the builder of the soaking pool.</p>

A

<p>Title to the soaking pool resides in the bidder, the fee simple owner of the land. Under the concept of fixtures, the soaking pool was converted from personalty into realty. The soaking pool is an accessory to the land and passes with the ownership of the land. (B) is incorrect because the document purporting to transfer the entrepreneur’s interest in the soaking pool to the buyer was insufficient to transfer real property. (C) is an incorrect statement of the law. (D) is incorrect because the soaking pool, as an accessory to the land, belongs to the owner of the land.</p>

45
Q

<p>An owner conveyed her parcel of land to her church “for the life of my son, and from and after the death of my said son to all of my grandchildren and their heirs and assigns in equal shares; provided that the church shall use the premises for church purposes only.” In an existing building on the property, the church immediately began to conduct religious services and other church activities. Subsequently, the church granted a construction company the right to remove sand and gravel from a one-half acre portion of the property on payment of a royalty. The construction company has regularly removed sand and gravel since and paid a royalty to the church. The church has continued to conduct religious services and other church activities on the property. All four of the living grandchildren of the owner, joined by a guardian ad litem to represent unborn grandchildren, instituted suit against the church and the construction company seeking damages for the removal of sand and gravel and an injunction preventing further acts of removal. There is no applicable statute.

Which of the following best describes the likely disposition of this lawsuit?

A The injunction and damages should be granted, because the interest of the church terminated with the first removal of sand and gravel.

B The injunction should be granted, and damages should be recovered but impounded for future distribution.

C The injunction should be granted, but damages should be denied because the owner and her son are not parties to the action.

D The injunction should be denied, but damages should be awarded.</p>

A

<p>Both an injunction and damages should be ordered. The church has a life estate pur autre vie, and a life tenant as a general rule is not entitled to consume or exploit natural resources on the property; this constitutes affirmative (voluntary) waste that injures the interests of the future interest holders. Any award of damages will be held until the class gift to the grandchildren closes at the son’s death. (A) is wrong because the church’s action did not terminate its interest. The “provided that” language creates a condition subsequent. An estate subject to a condition subsequent does not terminate automatically on the happening of the condition. To terminate, the grantor must exercise a right of entry, and here no right of entry was reserved. (C) is wrong because it is entirely unnecessary for the owner and her son to be parties, because neither of them has any interest in the land. The owner has given up her interest entirely, and the son is present in the conveyance only to serve as a measuring life for the life estate; he owns no interest in the land itself. (D) is wrong because the injury to the land is permanent and therefore should be prevented by an injunction.</p>

46
Q

<p>A small town had a municipal auditorium that all groups were permitted to use. Lately, a local preacher has begun to hold recruiting seminars for his religious cult at the auditorium. Sensing the displeasure of the voting public and fearing that the auditorium would become a mecca for fringe religious groups, the town council adopted the following ordinance: “Effective immediately, no religious groups will be permitted to use the municipal auditorium for meetings, speeches, or other public gatherings.” The preacher, who was having great success recruiting followers in the town, challenged the constitutionality of the ordinance in federal court.

Based on the above facts, should his challenge succeed?

A No, because the ordinance treats all religions equally.

B No, because continuing to allow religious groups to use the auditorium would violate the Establishment Clause of the First Amendment.

C Yes, because “religious groups” is an unconstitutionally vague term.

D Yes, because the town cannot show that the ordinance serves a compelling government interest.</p>

A

<p>The preacher’s challenge should succeed. Having created a forum generally open for use by all groups, the town must justify its exclusions therefrom under applicable constitutional norms. To justify discriminatory exclusion from a public forum based on the religious content of a group’s intended speech, the town must show that the ordinance is necessary to serve a compelling state interest, and that it is narrowly drawn to achieve that end, and nothing in the facts indicates that the town will be able to do so.

(B) suggests that the ordinance serves the compelling interest of maintaining separation of church and state. However, the former “equal access” policy did not offend the Establishment Clause. The former policy had a secular purpose (providing a forum in which citizens can exchange ideas); it avoided excessive entanglement with religion; and it did not have a primary effect of either advancing or inhibiting religion (permitting religious groups to use the auditorium would result in, at most, an incidental benefit).

Thus, (B) is incorrect. (A) is incorrect because an unjustified content-based exclusion of religious speech is not made more acceptable by virtue of the fact that it treats all religions equally.

(C) is incorrect because, although there is no authoritative constitutional definition of religion, it is unlikely that the term “religious groups” will be deemed to be impermissibly vague.</p>

47
Q

<p>An electric company is the sole distributor of electrical power in the city. The company owns and maintains all of the electric poles and equipment in the city in compliance with government standards. The electric company has had to replace insulators on its poles repeatedly because unknown persons regularly shoot at and destroy them, which causes the power lines to fall to the ground. On one of these occasions, a five-year-old child who had wandered out of his yard intentionally touched a downed wire, and was seriously burned.

If a claim on the child’s behalf is asserted against the electric company, will the child recover?

A Yes, if the electric company could have taken reasonable steps to prevent the lines from falling when the insulators were destroyed.

B Yes, because a supplier of electricity is strictly liable in tort.

C No, unless the electric company failed to exercise reasonable care to stop the destruction of the insulators.

D No, because the destruction of the insulators was intentional.</p>

A

<p>The child will recover under the conditions stated in (A). The electric company had notice that the insulators were being destroyed, causing the power lines to fall. Certainly, it was foreseeable that children, or anyone else, in the vicinity of a fallen line might be injured by such a dangerous condition. Consequently, the electric company had a duty to take reasonable steps, if possible, to prevent the lines from falling. Breach of this duty will lay the foundation for a recovery by the child.

(B) is simply an incorrect statement of the law; a supplier of electricity is not subject to strict liability.

(C) is incorrect because, even if the company took measures to stop the destruction of the insulators, it also had a duty to take precautions against the danger of falling power lines.

(D) is incorrect because the electric company had a duty to make its operations safe in light of the destruction of which it had notice. This is true even though the destruction was intentional. Criminal acts and intentional torts of third persons are not superseding forces where they are foreseeable, such as here.</p>

48
Q

<p>(*)
Acting on a hunch, a police officer went to a young woman’s apartment, broke in, and searched it. The officer found exactly what she was looking for under the woman’s bed: a sack filled with jewels. The attached note read, “Sweetheart, here are the goods from the estate heist. Your loving boyfriend.” It was well known in the community that the woman’s boyfriend was a jewel thief. The officer also knew that the estate of a local socialite had been burglarized three days ago. Just as the officer finished reading the note, the woman returned. The officer immediately placed the woman under arrest as an accessory to the estate burglary. Based on the evidence obtained from the woman’s apartment, a search warrant was issued for her boyfriend’s apartment. The search yielded burglar tools and more jewels from the estate. The boyfriend was immediately arrested and charged with the estate burglary. At the boyfriend’s trial for the estate burglary, his attorney files a motion to suppress the evidence consisting of the bag of jewels and note, the tools, and the jewels from the boyfriend’s apartment.

How should the court rule on the motion?
A Grant the motion as to the bag of jewels and note, but deny it as to the evidence found in the boyfriend’s apartment.

B Grant the motion, because all of this evidence is fruit of the poisonous tree.

C Deny the motion, because the police would have caught the boyfriend with the goods eventually.

D Deny the motion, because the police had a warrant to search the boyfriend’s apartment.</p>

A

<p>The court should deny the motion to suppress because the police had a warrant to search the boyfriend’s home. The boyfriend’s expectation of privacy extended only to his own home, which was searched under a warrant. He does not have standing to assert a Fourth Amendment claim regarding the search of his girlfriend’s apartment because her apartment was not his home, and he did not own it or have a right to possession of it. Thus, (A) is incorrect. Because the boyfriend cannot object to the search that provided the probable cause for the search of his apartment, (B) is also incorrect. (C) is not a valid justification because there is nothing to indicate that the seizure would fall under the “inevitable discovery” exception to the exclusionary rule.</p>

49
Q

<p>A general contractor about to bid on a construction job for an office building invited a carpenter and several others to bid on the carpentry work. The carpenter agreed to bid if the general contractor would agree to give the carpenter the job provided that his bid was lowest and the general contractor was awarded the main contract. The general contractor so agreed. The carpenter, incurring time and expense in preparing his bid, submitted the lowest carpentry bid. The general contractor used the carpenter's bid in calculating its own bid, which was successful.

Which of the following best supports the carpenter's position that the general contractor is obligated to award the carpentry subcontract to the carpenter?

A The carpenter detrimentally relied on the general contractor's conditional promise in preparing his bid.

B The carpenter gave consideration for the general contractor's conditional promise to award the carpentry subcontract to the carpenter.

C The general contractor has an implied duty to deal fairly and in good faith with all bidders whose bids the general contractor used in calculating its main bid to the building owner.

D The general contractor has an obligation to the owner of the building to subcontract with the carpenter because the carpenter's bid was used in calculating the general contractor's bid, and the carpenter is an intended beneficiary of that obligation.</p>

A

<p>(B) is correct. The carpenter's bid was consideration for the general contractor's promise to award the carpentry subcontract to the carpenter if his bid was the lowest and the general contractor was awarded the main contract. Thus, the general contractor and the carpenter formed a contract. Two elements are necessary to constitute consideration. First, there must be a bargained-for exchange between the parties; and second, that which is bargained for must constitute a benefit to the promisor or a detriment to the promisee. The carpenter and the general contractor agreed that the carpenter would supply a bid that the general contractor could use in its own bid. The carpenter's bid was bargained for and was a benefit to the general contractor, so it constitutes consideration sufficient to support the general contractor's conditional promise to award the subcontract to the carpenter. Conditional promises are enforceable, but the duty to perform does not become absolute until the condition has been met or is legally excused. The conditions in this contract were met-the carpenter's bid was the lowest and the general contractor was awarded the main contract. Thus, the general contractor is under a duty to perform his promise to award the subcontract to the carpenter.

(A) is not the best answer. The carpenter is seeking specific performance of their agreement so that he will be awarded the carpentry subcontract. If the carpenter uses a detrimental reliance or promissory estoppel argument, he would be conceding that he gave no consideration and there is no contractual obligation, but he should be awarded damages to prevent injustice. Courts will often limit damages under this theory to reliance damages, which could be much less than the value of the subcontract.</p>

50
Q

<p>At trial in an action for personal injuries suffered in a traffic accident, the plaintiff first calls the defendant as an adverse party. The plaintiff then calls a witness who was a passenger in the plaintiff's car but who also happens to be the defendant's former employer. On direct examination, the witness testifies to how the accident occurred and also expresses his opinion that the defendant is not a truthful person.

Which one of the following areas of questioning is most likely to be held beyond the proper scope of cross-examination?

A In letters to prospective employers, the witness has described the defendant as very honest and dependable.

B The defendant recently filed an action against the witness for breach of contract.

C The plaintiff's injuries were not as serious as the plaintiff is claiming.

D The witness has been falsifying his income tax returns.</p>

A

<p>(C) is correct. The scope of cross-examination is generally limited to: (i) matters brought up on direct examination; and (ii) matters concerning the witness's credibility (i.e., impeachment). Here, the witness's testimony on direct examination concerned how the accident happened and the defendant's character for untruthfulness; there is no indication that the witness testified about the plaintiff's injuries. Because the questioning does not concern the subject matter of the direct examination or the witness's credibility, it is most likely to be held beyond the proper scope of cross-examination. The other answer choices describe proper impeachment of the witness.

(A) is incorrect. A common method of impeachment is asking the witness about the witness's prior statements that are inconsistent with the witness's current testimony. Here the witness testified as to his opinion that the defendant is an untruthful person. Thus, it would be proper to cross-examine the witness about his inconsistent prior descriptions of the defendant's good character for honesty.

(B) is incorrect. A witness may be impeached with evidence that the witness is biased in favor of or against a party in the case. Here, the fact that the defendant recently filed an action against the witness for breach of contract shows that the witness may have a motive to testify against the defendant in this case.

(D) is incorrect. A party may impeach a witness by cross-examining the witness about the witness's specific acts of misconduct involving untruthfulness. Thus, it is proper to ask the witness about whether the witness falsified his tax returns.</p>

51
Q

<p>A mechanic sued his former employer in federal court, claiming that the employer had discharged him because of his age in violation of federal law. The employer answered, denying the claims and promptly moving for summary judgment. In support of the motion, the employer attached the mechanic's employment evaluations for the past three years, which rated his skills and performance as poor and culminated in a recommendation for his discharge.

What is the mechanic's best argument to defeat the summary judgment motion?

A The allegations in the complaint conflict with the mechanic's employment evaluations, raising a genuine dispute as to material facts.

B The employer cannot rely in his motion on matters outside the pleadings.

C The essential facts are unavailable to the mechanic and therefore discovery is required.

D The motion was filed before the close of discovery.</p>

A

<p>(C) is correct. If the mechanic (the nonmovant) shows by affidavit or declaration that he cannot present facts essential to justify his opposition to the summary judgment motion, Rule 56(d) authorizes him to ask the court to defer action or deny the motion to allow time to obtain affidavits or declarations or to take discovery. The employer moved for summary judgment right after answering and before any discovery. That timing would support defeating the summary judgment motion at this time.

(A) is incorrect. Under Rule 56(c), a party asserting that a fact is genuinely disputed must support the assertion by citing particular parts of the record, including affidavits or declarations, stipulations, or discovery materials. The mechanic cannot simply rely on the complaint allegations to rebut the employer's evidence but must support his factual position with his own evidence that a factual dispute exists. If he cannot do so, Rule 56(d) authorizes him to ask the court to defer action or deny the summary judgment motion to allow time to obtain affidavits or declarations or to take discovery.

(B) is incorrect. The function of a summary judgment motion is to allow additional evidence outside the pleadings to show that there is no genuine dispute of fact and that the movant is entitled to judgment as a matter of law. Rule 56(c)(1) enumerates the types of materials that the moving party may use to support a summary judgment motion, including documents. If the mechanic (the nonmovant) shows by affidavit or declaration that he cannot present facts essential to justify his opposition to the summary judgment motion, Rule 56(d) authorizes him to ask the court to defer action or deny the motion to allow time to obtain affidavits or declarations or to take discovery.

(D) is incorrect. The fact that a summary judgment motion is filed before the close of discovery does not require the court to deny it. Under Rule 56(b), a party may file the motion at any time until 30 days after the close of all discovery. The problem here is that the employer filed the motion before discovery commenced, thus providing the mechanic an argument to defer action or deny the motion under Rule 56(d).</p>

52
Q

<p>A builder contracted in writing to construct a small greenhouse on a homeowner's property for $20,000, payable upon completion. After the builder had spent $9,000 framing the greenhouse and an additional $1,000 for materials not yet incorporated into the greenhouse, the homeowner wrongfully ordered the builder to stop work.

The builder then resold the unused materials that he had already purchased for the greenhouse to another contractor for $1,000. At the time the homeowner stopped the work, it would have cost the builder an additional $5,000 to complete the project. The partially built greenhouse increased the value of the homeowner's property by $3,000.

In a suit by the builder against the homeowner, how much is the builder likely to recover?

A $3,000, the increase in the value of the homeowner's property.

B $10,000, the total cost expended by the builder at the time of the breach.

C $14,000, the total cost expended by the builder ($10,000) plus the builder's expected profit ($5,000), minus the loss avoided by the resale of the unused materials ($1,000).

D $15,000, the contract price ($20,000) minus the costs saved by the breach ($5,000).</p>

A

<p>(C) is correct. The builder is likely to recover $14,000. In a construction contract, if the property owner breaches the contract during construction, the builder is entitled to any profit he would have derived from the contract plus any costs he has incurred to date. If the builder has mitigated his damages, any losses that are avoided must be subtracted from this amount.

(A) is incorrect. In a construction contract, when the property owner breaches before the construction is completed, the builder's damages are not measured by the increase in value of the homeowner's property.

(B) is incorrect. The builder is also entitled to the profit he would have made if the contract had been performed.

(D) is also incorrect. The formula for awarding a builder damages for a breach during a construction contract can also be stated as the contract price minus the cost of completion, which would be $15,000. However, this answer fails to account for the $1,000 of damages the builder avoided by reselling the unused materials.</p>

53
Q

<p>A shop owner domiciled in State A sued a distributor in a federal district court in State A for breach of a contract. The shop owner sought $100,000 in damages for allegedly defective goods that the distributor had provided under the contract. The distributor is incorporated in State B, with its principal place of business in State C. The distributor brought in as a third-party defendant the wholesaler that had provided the goods to the distributor, alleging that the wholesaler had a duty to indemnify the distributor for any damages recovered by the shop owner. The wholesaler is incorporated in State B, with its principal place of business in State A.

The wholesaler has asserted a $60,000 counterclaim against the distributor for payment for the goods at issue, and the distributor has moved to dismiss the counterclaim for lack of subject-matter jurisdiction.

Should the motion to dismiss be granted?

A No, because the wholesaler's and the distributor's principal places of business are diverse.

B No, because there is supplemental jurisdiction over the wholesaler's counterclaim.

C Yes, because there is no diversity of citizenship between the distributor and the wholesaler.

D Yes, because there is no diversity of citizenship between the shop owner and the wholesaler.</p>

A

<p>B) is correct. There is supplemental jurisdiction because the claim for nonpayment for the goods in issue arises out of a common nucleus of operative fact as the plaintiff's (the shop owner's) claims, and the restrictions contained in the supplemental jurisdiction statute do not apply to these facts. This question tests on the restrictions placed on the use of supplemental jurisdiction when the case is in federal court under diversity jurisdiction and the use of supplemental jurisdiction is inconsistent with the requirements for diversity. The rule to remember here is this: The restrictions on the use of supplemental jurisdiction apply to plaintiffs only. Here, the defendant (the distributor) is the party using supplemental jurisdiction, so the restrictions do not apply. As a result, it does not matter that the defendant and third-party defendant are not diverse, and it does not matter that the amount in controversy requirement is not satisfied. (A) is an incorrect statement of the law. A corporation is a citizen of each state in which it is incorporated (only one in real life) and the one state in which it has its principal place of business. Since the wholesaler and the distributor are both incorporated in State B, complete diversity of citizenship does not exist. (C) is incorrect. Although it is a true statement of law, the answer ignores the potential for supplemental jurisdiction. (D) is incorrect. The shop owner is not asserting any claims against the wholesaler, so the lack of diversity between them is irrelevant.</p>

54
Q

<p>Eeven years ago, a man conveyed vacant land by warranty deed to a woman, a bona fide purchaser for value. The woman did not record the warranty deed and did not enter into possession of the land.
Five years ago, the man conveyed the same land to a neighbor, also a bona fide purchaser for value, by a quitclaim deed. The neighbor immediately recorded the quitclaim deed and went into possession of the land.

Two years ago, the neighbor conveyed the land to a friend, who had notice of the prior conveyance from the man to the woman. The friend never recorded the deed but went into immediate possession of the land.

The jurisdiction has a notice recording statute and a grantor-grantee index system.

If the woman sues to eject the friend, will the woman be likely to succeed?

A No, because the friend took possession of the land before the woman did.

B No, because the neighbor's title was superior to the woman's title.

C Yes, because the friend had notice of the conveyance from the man to the woman.

D Yes, because the woman, unlike the friend, took title under a warranty deed.</p>

A

<p>(B) is correct. Under a notice recording system, a subsequent bona fide purchaser ("BFP") prevails over a prior grantee who failed to record. A BFP is a purchaser who gives valuable consideration and has no notice of the prior grant. Notice includes actual, record, or inquiry notice. Also, under the shelter rule, a person who takes from a BFP will prevail against any interest that the transferor-BFP would have prevailed against. This is true even where the transferee had actual knowledge of the prior unrecorded interest. Here, the neighbor was a BFP. The woman had not recorded her deed and had not taken possession of the land; the neighbor gave value without any notice of the woman's claim. The neighbor's title was superior to the woman's title because of the recording statute. When the neighbor sold the land to the friend, the friend was protected under the shelter rule despite having actual knowledge of the woman's interest. Thus, the friend's title is also superior to that of the woman.

(A) is incorrect. Possession isn't required to establish title. The woman did not need to take possession before the friend to prevail; she needed to do something to put the first purchaser (the neighbor) on notice. If the woman had recorded her deed, the neighbor and the friend would not have had a valid claim to the property even if the woman had never taken possession of the property. Record notice would have put the neighbor on notice of her title, and he could not have been a BFP. The friend, taking from the neighbor, would not have been covered by the shelter rule. (If she did not record but took possession, that also would have put the neighbor on inquiry notice and he would not be a BFP.)

(C) is incorrect. Under the shelter rule, a person who takes from a BFP will prevail against any interest that the transferor-BFP would have prevailed against. This is true even where the transferee had actual knowledge of the prior unrecorded interest. Here, the neighbor was a BFP and the friend took from the neighbor. Thus, even though the friend had actual knowledge of the woman's deed, he is still protected by the shelter rule and has superior title to the property.

(D) is incorrect. The type of deed does not affect who has title. A quitclaim deed has the same effect as a warranty deed in terms of conveying title. The differences among the types of deeds have to do with remedies available against the grantor if title turns out to be defective. A warranty deed gives the grantee contractual promises with respect to title, and the quitclaim deed gives no promises</p>

55
Q

<p>A plaintiff sued the insurer of her home after the insurer denied coverage for water damage to the home allegedly caused by a frozen plastic pipe that burst. At trial, the insurer called as an expert witness an engineer, who testified that the pipe had burst because of age rather than freezing. On cross-examination, the engineer admitted that, five years earlier, he had been convicted of tax fraud, even though he had asserted that it was his accountant's error. In response, the insurer calls a witness, who is well acquainted with the engineer and his reputation, to testify that (1) in the witness's opinion, the engineer is a truthful person, and (2) the engineer's neighbors all describe him as a truthful person.

How much, if any, of the witness's testimony is admissible?

A All of the testimony is admissible to support the engineer's credibility.

B Only the portion concerning the engineer's reputation is admissible, because where both opinion and reputation evidence are available, only the latter is admissible under a rule of preference.

C Only the portion concerning the witness's opinion of the engineer's character, because the witness's reporting of the neighbors' comments is hearsay.

D None of the testimony is admissible, because it is collateral, having no bearing on the engineer's qualifications as an expert.</p>

A

<p>(A) is correct. Once a witness is impeached with evidence of his bad character for truthfulness, he may be rehabilitated with reputation and opinion evidence of his good character for truthfulness. Here the plaintiff has attacked the engineer's character for truthfulness by presenting evidence of his prior tax fraud conviction, so the insurer may now present evidence to rehabilitate the engineer. The witness's opinion and reputation testimony as to the engineer's good character for truthfulness is proper.

(B) is incorrect. Reputation and opinion evidence are both admissible to rehabilitate a witness and there is no rule of preference favoring one over the other.

(C) is incorrect. Although reputation evidence of character is hearsay in a sense-it is a compilation of out-of-court statements by those familiar with the person's reputation-there is a hearsay exception for reputation evidence of a person's character.

(D) is incorrect. Whether the engineer is a truthful person has bearing on his credibility, and the credibility of a witness is never a collateral matter.</p>

56
Q

<p>A seller contracted to manufacture 1,000 toasters for a buyer for a specified price. The contract contained a provision that clearly stated: "This contract may not be assigned, and any violation of this prohibition voids the contract." After the contract was signed, the seller informed the buyer that the toasters would be manufactured by a competitor of the seller. Citing the non-assignment provision, the buyer claimed that it was no longer bound by the contract. Toasters manufactured by the competitor were of equal quality to toasters manufactured by the seller.

Is the buyer bound by the contract?

A No, because "this contract may not be assigned" means that duties may not be delegated, and the seller delegated a duty.

B No, because the seller assigned a right despite the contractual prohibition.

C Yes, because even though the seller breached the contract, there are no damages since the competitor's toasters are of equal quality to the seller's toasters.

D Yes, because the non-assignment provision is not enforceable since public policy favors free assignment and delegation.</p>

A

<p>(A) is correct. A contract clause prohibiting the assignment of the contract will be construed as barring only the delegation of the assignor's duties. Here, the seller delegated its duty to manufacture the toasters to a competitor. This voided the contract, and the buyer is no longer bound.

(B) is incorrect. The seller delegated its duty to manufacture the toasters to a competitor. It did not assign its right to payment, which would have been permissible.

(C) is incorrect because, as stated above, the manufacturer's delegation of its duties rendered the contract void.

(D) is also incorrect. Nonassignment provisions will be enforced, but absent circumstances suggesting otherwise, a clause prohibiting the assignment of the contract will be construed as barring only the delegation of the assignor's duties.</p>

57
Q

<p>On the basis of scientific studies showing a causal relationship between the consumption of "red meat" (principally beef) and certain forms of cancer, a federal statute prohibits all commercial advertising of red meat products. The statute does not, however, restrict the sale of red meat products. Producers of red meat have challenged the statute as a violation of their free speech rights protected by the First Amendment.

Is the court likely to find the statute constitutional?

A No, because it does not serve a substantial government interest.

B No, because it is more extensive than necessary to serve the government interest in preventing certain cancers.

C Yes, because it does not affect speech protected by the First Amendment.

D Yes, because it serves a legitimate government interest in protecting public health.</p>

A

<p>(B) is correct. The regulation here involves commercial speech. Commercial speech is protected by the First Amendment, but the Court tests regulation of commercial speech under a special test. The Court first asks whether the speech is about a lawful activity and is truthful and not misleading. If these conditions are not satisfied, the speech has no protection. If they are satisfied, the regulation will be valid only if it (1) serves a substantial government interest, (2) directly advances that interest, and (3) is narrowly tailored to achieve that interest (that is, there is a reasonable fit between the means chosen and the ends sought). Here, the sale of red meat is allowed and so the producers are looking to advertise about a lawful activity. The regulation bans all commercial advertising. While the regulation serves a substantial government interest (that is, discouraging the consumption of a product linked to cancer), the regulation imposes a complete ban on advertisement. A complete ban will never be found to be narrowly tailored.

(A) is incorrect. The government will no doubt be found to have a substantial interest in preventing cancer.

(C) is incorrect. As indicated above, commercial speech is protected by the First Amendment.

(D) is incorrect. First, it is not enough merely that the interest served is legitimate - it must be substantial. And second, even if a legitimate interest were enough, that is only one prong of the test, and the regulation here would still fail because it is not narrowly tailored.</p>

58
Q

<p>A state law provides for an award of damages against anyone who publishes the name of a rape victim. Pursuant to that law, a woman sued a local newspaper in state court after the newspaper identified her as a rape victim.

The state trial and appellate courts rejected the claim, holding that the state law was invalid under both the state constitution and the First Amendment of the U.S. Constitution. The state supreme court affirmed, holding specifically: "We think that this well-intentioned law very likely violates the First Amendment of the federal Constitution. We need not, however, decide that issue, because the law assuredly violates our state constitution, which provides even greater protection to the right of the press to report the news." The woman petitioned for review in the U.S. Supreme Court.

Is the U.S. Supreme Court likely to review the state supreme court judgment?

A No, because the First Amendment prohibits the imposition of liability for the publication of truthful information.

B No, because the judgment of the state supreme court rests upon an adequate and independent state-law ground.

C Yes, because the Supremacy Clause does not permit a state to create rights greater than those conferred by the federal Constitution.

D Yes, because the U.S. Supreme Court's appellate jurisdiction extends to cases arising under federal law.</p>

A

<p>(B) is correct. The Supreme Court will hear a case from a state court only if the case turns on federal grounds. The Court will not hear a case if it finds adequate and independent nonfederal grounds to support the state decision. A ground is adequate if it is completely dispositive of the case, and it is independent if it does not rely on federal law. Here, the state court held that its decision rested solely on interpretation of its own constitution, and the state court decision completely disposed of the matter. Thus, it would be a waste for the Supreme Court to hear the case, because even if it decided the law was constitutional under the First Amendment, the law would remain unconstitutional under the state constitution. (A) is incorrect both because it is overbroad (for example, truthful publication of material in violation of another's copyright can be punished) and because the Court will not waste its time on a case where its decision will not change a result because the case rests on adequate and independent nonfederal grounds. (C) is simply not true. The Supremacy Clause does not prevent states from creating rights greater than those conferred by the federal Constitution - the federal Constitution is a floor and not a ceiling. It provides the minimal level of protection. States are free to provide greater protection. (D) is true but irrelevant. The Supreme Court does have appellate jurisdiction to hear cases arising under federal law, but the Court does not have to hear every case. Under the doctrine of strict necessity, it will not hear a case like this one, that rests on adequate and independent, nonfederal grounds.</p>

59
Q

<p>A toy collector had purchased 10 antique toys over the last several years and had had them restored by an expert in toy restoration. On June 1, the collector sent the 11th antique toy to the expert with a signed note that read: "Here is another toy for you to restore. As with all prior jobs, I will pay $500 for the work, but no more." On June 4, after receipt of the collector's June 1 note and the toy, the expert began restoring the toy. On June 6, the collector unexpectedly died. On June 7, unaware of the collector's death, the expert sent the collector a note that stated that the restoration work had begun on June 4. The following day, the expert learned of the collector's death.

Does a contract exist that binds the expert and the collector's estate?

A Yes, because the expert sent the June 7 note before learning of the collector's death.

B Yes, because the offer was accepted before the collector's death.

C No, because the collector died before the expert sent the June 7 note.

D No, because the offer lapsed when the collector died.</p>

A

<p>(B) is correct. The offer was accepted before the collector's death. The collector's letter was an offer. It created a reasonable expectation in the expert that the collector was willing to enter into a contract on the basis of the offered terms. Unless an offer specifically provides that it may be accepted only through performance, it will be construed as an offer to enter into a bilateral contract and may be accepted either by a promise to perform or by the beginning of performance. Here, the expert began performance and, thus, accepted the offer when he began restoring the toy on June 4 - two days before the collector died on June 6th.

(A) is incorrect. The collector's offer was accepted on June 4 when the expert began performance. Whether the expert knew of the collector's death on June 7 is not relevant. Indeed, if the expert had not already accepted the offer before the collector's death, the offer would have terminated on the collector's death by operation of law, even if the expert did not know of the death.

(C) is incorrect. The June 7 note was not an acceptance. The acceptance had already occurred on June 4 when the expert began performance by beginning to restore the toy.

(D) is incorrect. If either of the parties to a proposed contract dies prior to acceptance, the offer is terminated. Here the collector's offer had already been accepted at the time of the collector's death. Thus, there was no lapse.</p>

60
Q

<p>A man and his neighbor owned homes on adjacent lots in a subdivision. The subdivision's recorded restrictions did not prohibit detached storage sheds, and several homeowners in the subdivision had placed such sheds in their backyards. Because the man and the neighbor thought the sheds were unsightly, they both agreed in writing not to place detached storage sheds in their respective yards. Their agreement was drafted in recordable form and stated that it was enforceable by and against all assignees, heirs, and successors. The agreement was promptly recorded.
Three years later, the neighbor gave his home to his daughter. Shortly after moving into the home, the daughter learned of the restriction. She informed the man that she planned to put a detached storage shed in her backyard, claiming that the restriction was not enforceable against her.

Does the man have the right to enjoin the neighbor's daughter from placing a detached storage shed in her yard?

A No, because several homeowners in the subdivision have storage sheds in their yards.

B No, because there was no horizontal privity between the man and the neighbor.

C Yes, because the neighbor conveyed the home to the daughter by gift rather than by sale.

D Yes, because the restriction is binding on the daughter as a successor.</p>

A

<p>(D) is correct. A restrictive covenant can be enforced at law for damages (as a real covenant running with the land) or in equity for an injunction (equitable servitude). The requirements are different for each. Here the facts state that the man is seeking an injunction to prevent the neighbor's daughter from placing a shed in her yard. Thus, the man is trying to enforce the restrictive covenant as an equitable servitude. An enforceable equitable servitude requires that the covenanting parties intended the servitude to be enforceable against successors in interest, the successor in interest must have notice of the covenant, and the covenant must touch and concern the land. Here, the original parties clearly intended their successors to be bound because they stated in the agreement that it was enforceable by and against all assignees, heirs, and successors. The agreement was recorded. Thus, all successors would have constructive (record) notice of the covenant. For the benefit of a covenant to touch and concern the land, the promised performance must benefit the covenantee and their successors in their use and enjoyment of the benefited land. Here, the performance (not building a shed) benefited the man and his successors in their use and enjoyment of the land because they could enjoy their property without viewing sheds they consider unsightly. All requirements for an equitable servitude are met, and the main has a right to enjoin the neighbor's daughter.

(A) is incorrect because the question concerns enforcing an agreement between two parties; it does not involve a common development scheme. Thus, what the other neighbors do is irrelevant.</p>

61
Q

<p>A plaintiff filed an action in federal district court and served the defendant with the summons and complaint. The defendant moved to dismiss the complaint for failure to state a claim.

Instead of opposing the motion to dismiss, the plaintiff voluntarily dismissed the action and filed a new action, alleging the same claims but also addressing the pleading defects outlined in the defendant's motion to dismiss. The defendant then moved to dismiss the second action, and the plaintiff again voluntarily dismissed the second action instead of filing opposition papers.

The plaintiff then filed a third action, alleging the same claims but also including additional allegations that were responsive to the defendant's second motion. The defendant has moved to dismiss the third action; the plaintiff opposes the motion.

Is the court likely to grant the defendant's motion?

A No, because the plaintiff has promptly and diligently attempted to address the pleading defects.

B No, because the plaintiff voluntarily dismissed each previous action before the defendant filed an answer or moved for summary judgment.

C Yes, because the plaintiff failed to seek a court order dismissing the second action.

D Yes, because the plaintiff's previously dismissed actions asserting the same claims operate as an adjudication on the merits.</p>

A

<p>(D) is correct. A dismissal by notice is without prejudice unless the plaintiff has previously dismissed any federal or state court action on the same claim, in which case the dismissal by notice is with prejudice. (This is known as the "two dismissal rule.") Thus, the second dismissal by the plaintiff here operates as an adjudication on the merits. (A) is an incorrect statement of the law. The rule stated above applies even if the plaintiff has acted with diligence. (B) is incorrect as well. Although the filing of an answer or a motion for summary judgment cuts off the right of the plaintiff to voluntarily dismiss without leave of court, such actions do not affect the two dismissal rule. (C) also is incorrect. Leave of court is not required if the only pending motion is a motion to dismiss.</p>

62
Q

<p>A husband and wife acquired land as common law joint tenants with right of survivorship. One year later, without his wife's knowledge, the husband executed a will devising the land to his best friend. The husband subsequently died.

Is the wife now the sole owner of the land?

A No, because a joint tenant has the unilateral right to end a joint tenancy without the consent of the other joint tenant.

B No, because the wife's interest in the husband's undivided 50% ownership in the land adeemed.

C Yes, because of the doctrine of after-acquired title, or estoppel by deed.

D
Yes, because the devise to the friend did not sever the joint tenancy.</p>

A

<p>(D) is correct. Although as a general rule a joint tenant's interest is freely alienable during his or her lifetime without the consent of the other joint tenant, that interest cannot be devised in a will. In this case, on the death of the husband, the wife's interest in the joint tenancy immediately expanded and she became the sole owner of the land as the surviving joint tenant.

(A) is incorrect. As a general rule, a joint tenant's interest is freely alienable during his or her lifetime without the consent of the other joint tenant. However, as stated above, a joint tenant's interest cannot be devised in a will. In this case, on the death of the husband, the wife became the sole owner of the land as the surviving joint tenant.

(B) is incorrect. The doctrine of ademption applies only when an individual dies testate and attempts to devise land that the testator no longer owns. That doctrine is not applicable here because the wife became the sole owner of the land as the surviving joint tenant.

(C) is incorrect. The doctrine of after-acquired title, or estoppel by deed, applies when an individual attempts to convey title (usually by warranty deed) at a time when the individual does not have title to the land but later acquires title to the land. In this case, the husband had an interest in the land but did not have the power to devise it in a will, so the doctrine does not apply.</p>

63
Q

<p>A police officer was employed on a city's police force for 10 years. When the officer accepted the job, the city's employee benefit plan provided a death benefit to the spouse of any employee who died as a result of any job-related injury. Last year, the city amended its employee benefit plan to deny its death benefit in cases where the death "was caused by the employee's refusal to accept, for any reason other than its excessive risk to life or health, reasonably available medical care prescribed by a physician."

After this amendment took effect, the officer was shot while on duty. Because of a sincerely held religious belief, the officer refused to allow a prescribed blood transfusion and, as a result, died from loss of blood. When the officer's spouse applied for the death benefit, the city denied the application on the basis of the amendment to the employee benefit plan.

The officer's spouse has challenged the amendment, claiming that, as applied to the officer, it violated the officer's constitutional right to the free exercise of religion.

Is the court likely to find the amendment to the employee benefit plan constitutional as applied to the officer?

A No, because it effectively discriminates against a religious practice.

B No, because it violates the vested contractual rights of city employees who were hired before the amendment took effect.

C Yes, because it does not single out religious reasons for the denial of benefits and is a reasonable limitation on the award of such benefits.

D Yes, because it imposes a condition only on the award of a government benefit and does not impose a penalty on an individual's conduct.</p>

A

<p>(C) is correct. The court is likely to find the amendment to the employee benefit plan constitutional under the Free Exercise Clause. The Free Exercise Clause prohibits government from punishing someone (which includes denying a benefit) on the basis of the person's religious belief. However, the Supreme Court has held that the prohibition applies only to government conduct that targets conduct specifically because it is religiously motivated; a religiously neutral regulation that incidentally burdens religious conduct is valid, and the government need not include an exemption for religiously motivated instances of the regulated conduct. Here, the city amended its employee benefit plan to deny death benefits when death resulted from an employee's refusal to accept medical care. Nothing in the facts indicates the change was adopted in order to target the employee's religious beliefs; rather, the change seems to be a religiously neutral law that happened to burden the police officer's religious belief against accepting blood transfusions. Therefore, the amendment is constitutional. (A) is incorrect. Incidental discrimination against religious practices is not prohibited by the Free Exercise Clause. (B) is incorrect. This choice raises a Contract Clause issue while the officer's spouse challenged the benefit plan amendment under the Free Exercise Clause. Moreover, the Contract Clause merely limits the ability of states to enact laws that retroactively impair contract rights. Here, nothing indicates the city enacted any law or that the employee had a perpetual right to the death benefit the officer originally had. (D) is incorrect. Its reasoning is wrong - the Free Exercise Clause applies to denial of benefits as well as impositions of penalties.</p>

64
Q

<p>RAP : )
A woman owned land in fee simple absolute. The woman conveyed the land to a friend "for life," and when the friend died the land was to go to the woman's neighbor "and her heirs."
The neighbor died and in her duly probated will devised her entire estate to a local charity. If she had died intestate, her daughter would have been her only heir.

One year after the neighbor died, her daughter executed a quitclaim deed conveying any interest she might have in the land to the woman's friend.

The common law Rule Against Perpetuities is unmodified in the jurisdiction. There are no other applicable statutes.

Who has what interest in the land?

A The friend has a fee simple absolute, because his life estate merged with the remainder conveyed to him by the daughter.

B The friend has a life estate and the charity has a vested remainder, because the neighbor's interest was devisable.

C The friend has a life estate and the daughter has a vested remainder, because the deed from the woman created an interest in the neighbor's heirs.

D The friend has a life estate and the woman has a reversion, because the neighbor's remainder was void under the Rule Against Perpetuities.</p>

A

<p>(B) is correct. The grant here gave a life estate to the friend and a remainder in fee simple to the neighbor. It is a remainder because on the friend's death (the natural termination of the preceding estate), the neighbor has the right to possession. The remainder is vested because it was granted to an ascertainable person in being (the neighbor) and there were no conditions to prevent it from becoming possessory. A vested remainder is devisable by will; so when the neighbor devised her estate to the local charity, the charity took her vested remainder interest.

(A) is incorrect. Whenever the same person acquires all of the existing interests in land, present and future, a merger occurs, and that person then holds a fee simple absolute. Here, however, the daughter had nothing to convey to the friend. The woman conveyed a vested remainder in fee simple to the neighbor. That vested remainder is devisable, and the neighbor properly devised it by will to the local charity. The daughter, as an intestate heir, would have taken no interest in the property. A quitclaim deed conveys only the interest the grantor has, and in this case, that was nothing. Therefore, there was no merger. The daughter's deed conveyed no interest to the friend, and the friend has only his life estate.

(C) is incorrect. The phrase in the grant "and her heirs" indicates that a fee simple interest was being granted; it does not give the heirs any rights in the property. Thus, as discussed, the daughter acquired no interest in the remainder.

(D) is incorrect. The Rule Against Perpetuities states that no interest in property is valid unless it must vest, if at all, within 21 years after a life in being at the creation of the interest. Here, the neighbor's interest is already vested on creation; she has a vested remainder. Her estate is a vested remainder because it followed the natural termination of the preceding estate, and it was granted to an ascertainable person in being without any conditions to prevent it from becoming possessory. Because it is a vested remainder, it is valid under the Rule.</p>

65
Q

<p>A man became ill while at work and decided to go home early. When he entered his bedroom, the man saw his wife engaged in sexual intercourse with a neighbor. The man grabbed a gun from a dresser drawer and shot and killed the neighbor. He was later charged and prosecuted.

In a jurisdiction that follows the common law for homicide offenses, which crimes should the court instruct the jury on?

Press Enter or Space to submit the answer

A Murder and involuntary manslaughter.

B Murder and voluntary manslaughter.

C Murder, voluntary manslaughter, and involuntary manslaughter.

D Voluntary manslaughter and involuntary manslaughter.</p>

A

<p>(B) is correct. The court should instruct the jury on murder and voluntary manslaughter. Murder is the unlawful killing of another human being with malice aforethought. Malice aforethought exists if the defendant has one of the following states of mind: (i) the intent to kill, (ii) the intent to inflict great bodily injury, (iii) reckless indifference to an unjustifiably high risk to human life, or (iv) the intent to commit a felony. Here, the man grabbed a gun and shot and killed the neighbor. A jury could find that he had the intent to kill, or at least the intent to inflict great bodily injury. The court should therefore instruct the jury on murder. Voluntary manslaughter is a killing that would otherwise be murder but is distinguishable from murder by the existence of adequate provocation. Provocation will reduce a killing to voluntary manslaughter if it meets four tests: (i) the provocation must have been one that would arouse sudden and intense passion in the mind of an ordinary person such as to cause him to lose his self-control; (ii) the defendant must have in fact been provoked; (iii) there must not have been a sufficient time between the provocation and the killing for the passions of a reasonable person to cool; and (iv) the defendant in fact did not cool off between the provocation and the killing. Adequate provocation is frequently recognized in the case of one spouse discovering the other in bed with another person. Here, the man's actions meet the requirements for voluntary manslaughter. He caught his wife in bed with the neighbor, which would provoke an ordinary person and provoked the man himself. It appears that there were only a few seconds between the provocation and killing, which would be insufficient time for a reasonable person to cool, and nothing indicates that the man cooled off. Because the man's actions fit the requirements for voluntary manslaughter, the jury should be instructed on that crime. If the jury were to find that this killing did not qualify as voluntary manslaughter, the man would be guilty of common law murder. Thus the jury should be instructed on both crimes.

(A) is incorrect. The killing does not meet the requirements of involuntary manslaughter. There are two types of involuntary manslaughter: (i) a death caused by criminal negligence and (ii) a killing caused by an unlawful act, either a misdemeanor or a felony not included in felony murder. Here, there is no indication that the man was negligent; his actions appear to be intentional. Also, the man did not cause the neighbor's death while committing an unlawful act; he merely came home early from work. Because the killing does not fit into the definitions of involuntary manslaughter, the court should not instruct the jury on it.

(C) is incorrect. As discussed above, the killing does not meet the requirements for involuntary manslaughter, and the jury should not be instructed on it.

(D) is incorrect. As explained above, the court should instruct the jury on murder but not on involuntary manslaughter.</p>

66
Q

<p>A patient domiciled in State A sued a surgeon domiciled in State B in a federal court in State A, alleging claims for malpractice. The surgeon moved to dismiss the action for lack of personal jurisdiction. The court denied the motion and set discovery cutoff and trial dates.

The surgeon has appealed the denial of the motion.

Should the appellate court hear the merits of the surgeon's appeal?

A No, because the appellate court lacks jurisdiction over the appeal.

B No, because the district court's decision on jurisdiction is final.

C Yes, because a contrary appellate decision could terminate the action.

D Yes, because the surgeon's personal-jurisdiction challenge raises a constitutional question.</p>

A

<p>(A) is correct. With certain exceptions by rule or statute, only final judgments may be appealed. A final judgment is one that disposes of the whole case on its merits. Since the case was still pending after the denial of a motion to dismiss on the basis of a lack of personal jurisdiction, the judgment here was not a final judgment and thus was not appealable.

(B) is incorrect because an order determining personal jurisdiction may be appealed (and is not within the sole discretion of the district court), but the order must be "final" (that is, it must dispose of all claims by all parties).

(C) is an incorrect statement of law and the final order rule. The order must be "final" at the trial court level, not be potentially made final by an appellate court ruling.

(D) again is an incorrect statement of law. The final order rule may not be circumvented by the mere raising of a constitutional issue.</p>

67
Q

<p>Ten years ago, a couple bought a building and moved into its second-floor apartment with their teenage daughter. The couple operated a shoe store on the first floor of the building for many years. When the couple purchased the building, the area was predominantly rural and was zoned for nonresidential use. The municipality's zoning is cumulative.
Five years ago, the municipality rezoned the area to single-family residential use. The daughter was not aware of this change, since she was away at college.

Recently, the daughter inherited the building from her parents. The daughter immediately moved into the apartment and took over the operation of the shoe store on the first floor. The daughter has learned that a developer is planning to build a large residential community in the area surrounding her building.

The daughter has asked her lawyer for advice regarding her ability to continue operating the shoe store.

Should the lawyer advise the daughter that she can continue to operate her shoe store?

A No, because the nonconforming use of the building terminated when the daughter's parents died.

B No, but the municipality must pay her reasonable compensation for her loss resulting from the change in zoning.

C Yes, because the shoe store is a nonconforming use.

D Yes, because the zoning is cumulative and the building is also used for single-family residential purposes.</p>

A

<p>(C) is correct. A cumulative zoning ordinance creates a hierarchy of uses of land, and land that is zoned for a particular use may be used for the stated purpose or for any higher use. A residential use is higher than a nonresidential use. Here, the building was in an area originally zoned for nonresidential use. The daughter and her parents used the property for a business and their residence. This was appropriate under the cumulative zoning ordinance as the family's uses met or exceeded the zoned use. Later, the area was rezoned for single-family residential use, which is a higher use than the shoe store. However, a use that exists at the time of passage of a zoning ordinance and that does not conform cannot be eliminated at once. Generally, the nonconforming use may continue indefinitely, but any change in the use must comply with the zoning ordinance. Because the shoe store existed at the time of the rezoning, the daughter may continue to operate the shoe store as a nonconforming use.

(A) is incorrect. Generally, the nonconforming use may continue indefinitely. The key is the use of the property and not the ownership. The parents' deaths do not affect the zoning status of the building. Thus, the daughter may live in the building and operate the shoe store.

(B) is incorrect. The zoning power is limited by the "no taking without just compensation" clause of the Fifth Amendment. Nevertheless, zoning is generally not a "taking" unless it amounts to a physical appropriation of the property or denies the owner of all economic use, which would not be the case here (the daughter could use the property as a single-family residence). Because the property is a nonconforming use, she will be allowed to continue that use.

(D) is incorrect. A cumulative zoning ordinance creates a hierarchy of uses of land, and land that is zoned for a particular use may be used for the stated purpose or for any higher use. A single-family home is a higher use than an apartment building or a commercial use such as a shoe store. Because the shoe store is a lower use than a single-family residence, it would not be allowed in this area. Nor would the daughter's apartment above the store. The store and apartment are allowed only because they are a nonconforming use.</p>

68
Q

<p>A buyer sent a signed letter to a seller that stated: "Ship 100 boxes of nails at $3 per box, the price quoted in your circular." The seller mailed the buyer a signed form acknowledgment that agreed to the buyer's terms and stated on the reverse side: "Disputes regarding quality shall be arbitrated." The buyer did not reply to the seller's acknowledgment, and the seller shipped the nails. When the buyer received the nails, it found their quality to be unsatisfactory and sued the seller for breach of warranty. The seller has asked an attorney whether the parties' contract requires arbitration of the buyer's claim.

What is the best advice the attorney can provide?

A A contract was formed pursuant to conduct when the buyer received the nails, and a court would exclude the arbitration provision from the contract.

B A contract was formed when the seller mailed its acknowledgment, and the arbitration term became part of the contract.

C A contract was formed when the seller mailed its acknowledgment, and the court must decide whether the arbitration term should be excluded as a material alteration of the contract.

D No contract exists, because the arbitration term in the seller's acknowledgment created a counteroffer that the buyer never accepted.</p>

A

<p>(C) is correct. This is a contract for the sale of goods (nails) so Article 2 of the UCC applies. Under Article 2, the proposal of additional or different terms by the offeree in a definite and timely acceptance does not constitute a rejection and counteroffer, but rather is effective as an acceptance, unless the acceptance is expressly made conditional on assent to the additional or different terms. If both parties to the contract are merchants, the additional terms in the acceptance will be included in the contract unless they materially alter the original terms of the offer; the offer expressly limits acceptance to the terms of the offer; or the offeror has already objected to those terms, or objects within a reasonable time after notice of the terms is received. Whether an alteration is material is a fact question. Here, the seller's acknowledgment contained an additional term - but was not made conditional on its acceptance. Therefore, a contract was formed. The seller is clearly a merchant, and given the size of the order, it appears the buyer is a merchant too. Thus, the court must decide whether the arbitration term should be excluded as a material alteration of the contract.

(A) is incorrect. The contract was formed when the seller mailed the buyer the signed acknowledgment form.</p>

69
Q

<p>A valid treaty between the United States and a foreign country provides for the elimination of all tariff barriers between the two countries. It authorizes the president of either country to issue a proclamation nullifying any state or local laws in that country that have the effect of impeding imports from the other country.

The foreign country uses the metric system of measurement, and thus all goods produced there and exported to the United States are packaged in metric sizes, such as liters and kilograms. A law of a state in the United States requires all goods sold in that state to be packaged in traditional American sizes, such as quarts or pounds. Because the state law substantially impedes imports from the foreign country, the President of the United States has issued a proclamation nullifying the state law pursuant to the treaty.

Is the President's proclamation valid?

A No, because the Constitution vests in Congress the exclusive authority to specify binding legal standards for weights and measures, and the President therefore lacks constitutional authority for the proclamation.

B No, because the principles of federalism embedded in the Constitution prohibit the President from taking action to invalidate a state law.

C Yes, because it is authorized by a valid treaty of the United States and is not prohibited by any provision of the Constitution and, therefore, is the supreme law of the land.

D Yes, because the President has inherent authority to nullify any state law that substantially impedes commerce between the United States and another country.</p>

A

<p>(C) is correct. The U.S. Constitution, federal laws, and treaties are the supreme law of the land; state laws that are in conflict with the supreme law are invalid. Here, a valid treaty gives the President the power to nullify state and local laws that impede imports between the signatory nations. There is nothing in the Constitution that would prohibit such a law; indeed, the Constitution gives Congress the power to regulate commerce both among the states and with other nations. (A) is incorrect as Congress is free to delegate its authority, and here it has delegated its authority to the President. Moreover, if (A) is true and Congress has exclusive authority to specify weights and measures, the state law setting required weights and measures would be invalid anyway. (B) is simply not true. As stated above, the Constitution, federal laws, and treaties are the supreme law of the land. These can certainly give the President authority to invalidate state laws that interfere with federal objectives that are within the powers of Congress, such as objectives under the Commerce Clause. (D) is incorrect because it is overbroad. The President has no such inherent power. It would be for the courts to decide if a state law impedes commerce between the United States and another country.</p>

70
Q

<p>A man has sued a police officer, alleging that the officer violated the man's civil rights by using excessive force while arresting him. At trial, the officer admits having hit the man in the head with the butt of his gun, but contends that the force was necessary, because the man was resisting arrest. In support of his contention, the officer seeks to introduce evidence that the man had resisted arrest on three prior occasions during the last 10 years.

Is this testimony regarding the man's conduct during the three prior arrests admissible?

A No, because evidence of the prior incidents constitutes impermissible character evidence.

B No, because the officer has not shown that the man was convicted in connection with the prior incidents.

C Yes, because the incidents in question are relevant evidence of the man's propensity for violence.

D Yes, because the incidents in question are sufficient to constitute a habit.</p>

A

<p>(A) is correct. Evidence of a person's other acts is generally inadmissible to show how a person probably acted on a particular occasion. Although such evidence may be admitted if relevant for an independent purpose, here the evidence only tends to show that the man has a propensity to resist arrest. The officer is seeking to show that because the man resisted arrest on the prior occasions, he is more likely to have resisted arrest during the events of this case. Consequently, it is inadmissible character evidence. (B) is incorrect. The evidence would be inadmissible to show the man's propensity to resist arrest even if the man had been convicted of the prior acts. And even though evidence of certain prior convictions may be introduced to impeach a witness, there is no indication that the man has testified at this trial. (C) is incorrect. A person's other acts are generally not admissible to show the person's propensity to act a certain way. Thus, the evidence cannot be admitted for this purpose. (D) is incorrect. Habit evidence is evidence of a person's regular response to a specific set of circumstances. It is admissible to prove that the person acted in accordance with the habit on a particular occasion. However, resisting arrest three times over a span of 10 years is not frequent enough to constitute habit.</p>

71
Q

<p>A horse breeder offered to sell a colt to his neighbor and they agreed on a purchase price. The horse breeder subsequently received a letter from the neighbor thanking him for the sale and summarizing their agreement. The letter contained the neighbor’s alleged signature. When the horse breeder attempted to set up transfer of the colt, the neighbor denied that she agreed to purchase it. In a breach of contract action against the neighbor, the horse breeder offers into evidence the letter. The horse breeder testifies that he is familiar with the neighbor’s handwriting and recognizes the signature on the letter as being hers.

Assuming appropriate objection by the neighbor, who claims that she did not sign the letter, how should the trial court rule on the admissibility of the letter?

A Exclude the letter for lack of foundation because lay opinion testimony regarding handwriting identification is not admissible.

B Exclude the letter unless its authenticity is established by a preponderance of the evidence.

C Admit the letter as authentic and instruct the jury accordingly.

D Admit the letter but instruct the jury that it is up to them to decide whether the letter is authentic.</p>

A

<p>[where there is an issue of authenticity, the jury is the one that has to decide]

The court should admit the letter and instruct the jury that it is up to them to decide whether the letter is authentic. Before a writing may be received in evidence, it must be authenticated by proof showing that the writing is what the proponent claims it is. All that is necessary is proof sufficient to support a jury finding of genuineness. The authenticity of a document is a preliminary fact to be decided by the jury. Here, the horse breeder’s testimony that he is familiar with the neighbor’s handwriting and that he recognizes the signature on the letter to be that of the neighbor is sufficient to support a jury finding of genuineness. Thus, the letter should be admitted and authenticity should be left to the jury to decide.

(C) is wrong because, as noted above, where there is a dispute as to the authenticity of a document, the issue of authenticity is a fact determination for the jury, not the judge, to decide.</p>

72
Q

<p>(*) A homeowner, a citizen of State A, hired an electrician, a citizen of State B, to fix the wiring in her basement and hired a gas worker, also a citizen of State B, to install a new gas stove in her kitchen. Unfortunately, the home caught fire and burned down while they were both working on their separate jobs. The homeowner sued the gas worker for negligence in federal court in State A, seeking $100,000. The homeowner promptly served the gas worker, and the gas worker timely filed an answer with the court. One month after filing the answer, the gas worker moved to file and serve a third-party complaint against the electrician, alleging that the electrician was the sole cause of the accident.

Which of the following arguments is most likely to achieve the electrician's goal of dismissal of the third-party complaint?

A The gas worker's motion for leave to file a third-party complaint is untimely and thus should be denied as a matter of law.

B The court does not have subject matter jurisdiction over the third-party complaint because the electrician's claim and the gas worker's claim do not arise from a common nucleus of operative fact.

C The gas worker's claim against the electrician is not a proper third-party claim.

D Dismissing the gas worker's claim will not impede his ability to protect his rights in a separate action.</p>

A

<p>The electrician's best argument is that the gas worker's claim against the electrician is not a proper third-party claim. Under Rule 14, a defendant may assert a third-party claim against "a nonparty who is or may be liable to it for all or part of the claim against it." In other words, a third-party claim must be a derivative claim; the third-party plaintiff must be seeking indemnification or contribution from the third-party defendant. Here, the gas worker's claim is not that the electrician must indemnify him or that the electrician is a joint tortfeasor who may be jointly liable under principles of contribution. Rather, the gas worker is alleging that he (the gas worker) is not liable and that the electrician is. Because the claim is not derivative, it is not properly asserted as a third-party claim under Rule 14.

(A) is incorrect. A defendant may serve a third-party complaint as of right within 14 days of serving his original answer. Thereafter, he must make a motion to serve the complaint, and it is within the trial court's discretion whether to grant or deny the motion. Here, it is unlikely that a court would deny a defendant's motion to serve a third-party complaint at such an early stage of the proceeding.</p>

73
Q

<p>Based on recommendations of a state commission studying the effect of pornographic films on violent criminal activity, a state adopted legislation banning films intended for commercial distribution that appealed as a whole to the prurient interest in sex of the average person in the community, portrayed sex in a patently offensive way to citizens of the state, and which a reasonable person in the United States would find had no serious literary, artistic, political, or scientific value.

In ruling on a constitutional challenge to the legislation from a film distributor in the state who was convicted of distributing films in violation of the legislation, will the federal court likely find the legislation to be constitutional?

A Yes, because it uses a national “reasonable person” standard for determining the social value of the work.

B Yes, because it uses a statewide standard rather than a community standard for determining whether the material is patently offensive.

C No, because it uses a statewide standard rather than a national standard for determining whether the material is patently offensive.

D No, unless the court finds that the legislation is necessary to advance the state’s compelling interest in reducing violent criminal activity.</p>

A

<p>The court will likely find the legislation to be a constitutional regulation of obscenity. Obscenity, which is not protected speech under the First Amendment, is defined by the Supreme Court as a description or depiction of sexual conduct that, taken as a whole, by the average person, applying contemporary community standards, appeals to the prurient interest in sex, portrays sex in a patently offensive way, and—using a national reasonable person standard—does not have serious literary, artistic, political, or scientific value. Thus, the legislation here is constitutional because it uses a reasonable person standard, rather than a community standard, for determining the value of the work.

(B) is incorrect because while a statewide standard for determining whether the material is patently offensive is permissible, it is not mandatory. A state may use a “community standard” for making this determination.

(C) is incorrect because, again, a statewide standard for determining whether the material is patently offensive is permissible. Only the “social value” element of the obscenity test requires a national standard.

(D) is incorrect because the legislation is valid regardless of whether it is necessary to achieve the state’s compelling interest in reducing violent crime. Speech that falls within the definition of obscenity is unprotected speech; the government does not need a specific compelling interest to ban it.</p>

74
Q

<p>A father was angry at his son’s coach because the coach would never let the son into a game. In order to exact revenge, the father decided to plant an incendiary device on the coach’s front porch. The father believed the device would start a fire that would destroy the coach’s home and perhaps injure him as well. However, the father made a mistake while assembling the incendiary device, and it was impossible for the device to do any harm. When the device went off, it did nothing more than produce a foul odor.

If the father is charged with attempted murder and attempted arson in a common law jurisdiction, which of the following decisions is most likely to be reached by the court?

A The father is guilty of attempted murder and attempted arson.

B The father is guilty of attempted murder, but he is not guilty of attempted arson.

C The father is not guilty of attempted murder, but he is guilty of attempted arson.

D The father is not guilty of attempted murder or attempted arson.</p>

A

<p>The father lacked the specific intent to kill that is required for attempted murder. However, the circumstances surrounding the “incendiary device” constitute factual impossibility and will not afford the father a defense to attempted arson. Criminal attempt is an act that, although done with the intention of committing a crime, falls short of completing that crime. To be guilty of attempt, the defendant must have the intent to perform an act and obtain a result that, if achieved, would constitute a crime. Regardless of the intent that would suffice for the completed offense, attempt always requires a specific intent to commit the target offense. Also, the defendant must have committed an act beyond mere preparation for the offense. Here, to be guilty of attempted murder, the father must have had the specific intent to kill his son’s coach, even though the intent to inflict great bodily injury would be sufficient mens rea for murder. However, the facts indicate that the father intended at most only to injure the coach rather than kill him. Thus, the father cannot be guilty of attempted murder. However, the father did intend to burn the coach’s home; therefore, he had the specific intent to commit arson by means of placing an incendiary device on the coach’s porch, and his placing the device was an act beyond mere preparation for this crime. Although the device could not have actually burned the coach’s house, it is no defense to attempt that it would have been impossible for the defendant to complete his plan. This is factual impossibility and is not a defense. Thus, the father is guilty of attempted arson.

(A) and (B) are incorrect because the father did not have the specific intent to kill. (D) is incorrect because the father is guilty of attempted arson, as explained above.</p>

75
Q

<p>A husband and a wife were arrested by federal agents and charged with distributing obscene materials through the United States mails. When called before a grand jury, the wife refused to say anything, invoking her Fifth Amendment right to be protected from compelled self-incrimination. The husband was terrified of the grand jury and readily admitted under questioning that he sent obscene matter through the mail. He also incriminated his wife in the illegal activity. The thought of a trial and a prison term drove the husband over the edge, and he committed suicide two days before his trial was to begin. A month later, the wife was put on trial in federal district court. The federal prosecutor seeks to introduce a transcript of the husband’s grand jury testimony into evidence against the wife. The defense attorney objects.

How should the court rule on the admissibility of the grand jury transcript?

Press Enter or Space to submit the answer

A Admissible, as a vicarious statement of an opposing party.

Incorrect
B Admissible, as former testimony.

C Inadmissible, because the wife can invoke the testimonial privilege, even though her husband is now deceased.

Correct
D Inadmissible, because the husband’s testimony was not subject to cross-examination.</p>

A

<p>The grand jury transcript is not admissible because the husband’s testimony was not subject to cross-examination. The husband’s testimony was hearsay because it was an out-of-court statement offered to prove the truth of the matter asserted. [Fed. R. Evid. 801(c)] If a statement is hearsay, and no exception to the rule is applicable, the evidence is inadmissible. [Fed. R. Evid. 802] Under the former testimony exception to the hearsay rule, the testimony of a now unavailable witness given at another hearing is admissible in a subsequent trial as long as there is a sufficient similarity of parties and issues so that the opportunity to develop testimony or cross-examine at the prior hearing was meaningful. [Fed. R. Evid. 804(b)(1)] The party against whom the former testimony is offered must have had the opportunity to develop the testimony at the prior proceeding by direct, cross-, or redirect examination of the declarant. Thus, the grand jury testimony of an unavailable declarant is not admissible as former testimony against the accused at trial. This is because grand jury proceedings do not provide the opportunity for cross-examination. Therefore, because the husband’s testimony was in front of the grand jury and was not subject to cross-examination, it is inadmissible as hearsay. (</p>

76
Q

<p>A hardware store ordered 200 cans of wood stain in various shades. The written contract between the store and manufacturer provided that 100 cans of stain would be delivered on April 30, and the remaining 100 cans would be delivered on June 30. Payment would be due at the time of each delivery. The first shipment arrived on April 30. Sales of the stain were brisk, but 25 customers almost immediately returned their stain, complaining that it was not the color indicated on the can. The store owner called the manufacturer and informed it of the problem. The manufacturer truthfully told the owner that they had had a small problem with their labeling machine and a few cans in the store owner's lot must have been mislabeled before they caught the problem. The manufacturer offered to replace all 100 cans from the original order. The store owner refused the offer and told the manufacturer not to deliver the second lot, because he could no longer trust the manufacturer. The owner was very sensitive to the hardware store's good reputation, which he felt was harmed by this incident.

If the manufacturer brings a claim of breach regarding the second shipment which was due on June 30, how will the court likely rule?

A The buyer had the right to cancel the second shipment, because of legitimate fears that it would contain the same defects as the first shipment.

B The buyer had the right to cancel the second shipment, because the first delivery was defective.

C The buyer did not have the right to cancel the second shipment, because the defects in the first shipment did not substantially impair the value of the entire contract.

D The buyer did not have the right to cancel the second shipment, because he failed to make a demand upon the manufacturer for adequate assurances that the second shipment would be free of defects.</p>

A

<p>The buyer did not have the right to cancel the second shipment, because the defects in the first shipment did not substantially impair the value of the entire contract. This case involves an installment contract, i.e., the contract authorizes or requires deliveries in separate lots, and the sale of goods, so Article 2 of the UCC applies. Under Article 2, a buyer may declare a total breach of an installment contract only if the defect substantially impairs the value of the entire contract. [UCC §2-612] The problem with the first shipment of the stain was discovered and corrected by the manufacturer. The manufacturer offered to cure the defect in the first shipment. In whole, the defect in the first shipment did not substantially impair the value of the entire contract. (A) is incorrect because legitimate fears, alone, are not enough to justify anticipatorily repudiating a contract, as the buyer did here. Anticipatory repudiation occurs when a promisor, prior to the time set for performance of his promise, indicates that he will not perform when the time comes. If, as here, the promisor is unsure of whether the other party will fulfill his contract obligations, the promisor may seek adequate assurances that performance will be forthcoming. In this case, the manufacturer already provided the buyer with such assurances, which fell on deaf ears. Because the manufacturer assured the buyer and, more importantly, had already corrected the problem at its factory before the second shipment was due, the store owner's fears would not justify canceling the second shipment.

(B) is incorrect because, as discussed above, the defects in the first delivery would not warrant cancellation of the entire installment contract unless they substantially impaired the value of the entire contract, which they did not.

(D) is incorrect because, as explained, the manufacturer had already given adequate assurances to the buyer.</p>

77
Q

<p>A seller put her house and lot on the market for $200,000. After receiving several offers within $5,000 of her asking price, the seller entered into a contract to sell the house and lot to a buyer for $200,000. The contract provided that the buyer put up $4,000 in earnest money, which the seller could treat as liquidated damages unless:

The seller fails to tender marketable title to the buyer by the agreed-upon closing date, the seller commits a material breach of this contract, or the buyer dies prior to the closing date, in which case the earnest money shall be reimbursed to the buyer's estate.
The contract was signed on July 24, and the closing date was set for September 12.

On August 5, the buyer was seriously injured in an accident. On September 10, the buyer was released from the hospital in a wheelchair. He determined that a ranch-style house would make his life much more bearable, but the seller's home was two stories. The buyer asked the seller to cancel the contract and to refund the $4,000 earnest money. The seller refused. The buyer did not appear on the closing date. On September 16, the seller contracted to sell the home to a purchaser for $198,000. The closing occurred as planned on October 20. The buyer files suit against the seller, praying for a refund of the $4,000 earnest money.

How much is the buyer likely to recover?

A The entire $4,000, because the buyer had a justified medical reason for his failure to perform.

B $2,000, because the diminution in value of the property was only $2,000.

C $2,000 less any of the seller's out-of-pocket costs involved in remarketing the home.

D Nothing, because at the time the contract was entered into, $4,000 represented a reasonable estimate of damages in the event of breach.</p>

A

<p>The buyer will most likely recover nothing because, at the time of the contract, $4,000 represented a reasonable estimate of damages in the event of breach. When a sales contract provides that a seller may retain the buyer's earnest money as liquidated damages, courts routinely uphold the seller's retention of the money upon breach if the amount appears reasonable in light of the seller's anticipated and actual damages. Many courts uphold retention of earnest money of up to 10% of the sales price without inquiry into its reasonableness. In this case, the earnest money represented 2% of the purchase price. Given the fact that the seller had received other offers within $5,000 of the price offered by the buyer, $4,000 would be a reasonable estimate of damages if the seller were forced to accept another offer.

(A) is wrong because the fact that the buyer had a good reason for not performing does not change the fact that he is in breach. The contract is not impossible for the buyer to perform; it is just not as attractive a purchase as it was before the accident. He cannot escape liability on this basis. (B) and (C) are wrong because if there is a valid liquidated damages clause, it will be enforced and actual damages are irrelevant. (If the liquidated damages clause were not enforceable,

(C) would be a better choice than
(B) because the seller would be entitled to her expenses in remarketing the property.) Thus, because the liquidated damages clause is enforceable, the buyer will not be able to recover any of the $4,000 he paid as earnest money.</p>

78
Q

<p>A wealthy sportsman purchased a large old wooden sailing ship for $200,000. Although the boat was a classic, the sportsman wanted it to be modernized and made more comfortable. To that end, the sportsman entered into a written contract with a marine architect-engineer to draw up and then execute the modernization plans, for $7,500.

At the time the parties entered into the agreement, the sportsman told the architect-engineer that his modernization plan would be subject to the approval of the sportsman's sister, that they would, in fact, have no deal unless the plans meet with her approval. The architect-engineer agreed to this. He finished his drawings and submitted them to the sportsman, who was enthusiastic about the designs. The sportsman's sister, a famous yachtswoman, was engaged in a trans-Pacific yacht race at the time and was not expected home for a number of weeks. Cheered by the sportsman's enthusiasm, the architect-engineer went ahead and modernized the ship according to his designs. When he finished the work, he submitted a bill to the sportsman, who refused to pay, pointing out that his sister had never approved the designs.

If the architect-engineer sues the sportsman, which of the following issues of contract law is most likely to be decisive in determining the outcome of the case?

A Statute of Frauds.

B Parol evidence rule.

C Rules of construction.

D Conditions precedent.</p>

A

<p>Approval of the modernization plans by the sportsman's sister is a condition precedent because without such approval the parties have no agreement. Where there is an oral condition precedent, evidence of the condition falls outside the parol evidence rule. The parol evidence rule provides that where the parties to a contract express their agreement in a writing with the intent that it embody the final expression of their bargain, any other expressions, written or oral, made prior to the writing, as well as any oral expressions contemporaneous with the writing, are inadmissible to vary the terms of the writing. Certain forms of extrinsic evidence are deemed to fall outside the scope of the parol evidence rule. For instance, a party to a written contract can attack the validity of the agreement. One way of doing so is by asserting that there was an oral agreement that the written contract would not become effective until the occurrence of a condition. Such a condition would be deemed a condition precedent to the effectiveness of the agreement, and evidence of the condition will be freely offered and received. Here, the sportsman and the architect-engineer have entered into a written agreement that apparently embodies the full and final expression of their bargain. However, the sportsman's statement at the time of entering into the agreement indicates quite clearly that the parties had no agreement absent the approval of his sister, and the architect-engineer agreed with this statement. Thus, there is an oral agreement that the written contract would not become effective until the occurrence of a condition precedent. As discussed above, evidence of this oral condition does not come within the purview of the parol evidence rule and is therefore admissible. Consequently, the sportsman can assert the nonoccurrence of a condition precedent as a way to avoid liability on the contract.

(B) is incorrect because, as explained above, the nature of the oral agreement takes it outside the scope of the parol evidence rule. Therefore, the rule will not be decisive in determining the outcome of this case.</p>

79
Q

<p>Acting on information from reliable informants that drugs were being sold by residents at a certain fraternity house, the police obtained a search warrant that entitled them to search the entire premises for illegal narcotics. The police arrived at the house when a party was in progress and were admitted to the house by the fraternity president after showing the warrant. Officers proceeded to search the house. In an upstairs bedroom, they found a young woman who was a guest of a fraternity member sleeping on the bed. No one else was in the room. The police found a footlocker under the bed and opened it, finding a variety of illegal drugs. The police then awakened the woman and seized her purse from her. They found a small quantity of marijuana in the purse. The woman was charged with a drug possession offense. At her trial, the prosecution seeks to admit the marijuana seized from her purse over the objection of her attorney.

Should the court admit the marijuana?
A Yes, because the footlocker was within the woman's reach.

B Yes, because the woman was present in a room where drugs were found.

C No, because the woman had no possessory interest in the premises.

D No, because the police had no reason to believe that the woman had drugs on her person.</p>

A

<p>The court should not admit the marijuana into evidence because it was obtained as a result of an unreasonable search of the woman. Under the exclusionary rule, evidence obtained in violation of a defendant's Fourth Amendment rights is not admissible to establish the guilt of the defendant at trial. The Fourth Amendment protects against unreasonable searches and seizures by criminal law enforcement agents. To have a protected Fourth Amendment right, a person must have a reasonable expectation of privacy with respect to the place searched or the item seized. For a search based on a search warrant to be constitutionally valid, the warrant must be based on probable cause and must describe with reasonable precision the place to be searched and the items to be seized. A search warrant does not authorize the police to search persons found on the premises who are not named in the warrant. However, if the police have probable cause to arrest a person discovered on the premises to be searched, they may conduct a warrantless search of her incident to the arrest. If a person is not named in the warrant and circumstances justifying an arrest of that person do not exist, the police may search her for the objects named in the search warrant only if they have probable cause to believe that she has the named objects on her person. Here, the search warrant was issued on the basis of information from reliable informants, and it stated precisely the premises (the fraternity house) to be searched and the items (illegal narcotics) to be seized. Thus, the warrant is valid.

However, the search warrant (which did not name the woman) did not authorize the police to search the woman's purse. The search cannot be justified as incident to a valid arrest because: (i) the police searched the purse before they arrested the woman, and (ii) the police did not have sufficient probable cause to arrest the woman prior to searching the purse. Because the police had no reason to believe that the woman had drugs on her person, they cannot successfully claim that they were searching for the drugs mentioned in the warrant. The woman can challenge the search because she had a possessory interest and a reasonable expectation of privacy in her purse, which was the object of the search. Thus, because the marijuana was seized pursuant to an unreasonable search in violation of the Fourth Amendment, it must be excluded from evidence</p>

80
Q

<p>At a products liability trial, a critical issue is whether the temperature was below freezing on January 16. A local man who works for a civil engineering firm is also an avid amateur meteorologist. One of the man's weather detection instruments in his backyard records temperature by markings from a stylus on a round barograph. The man's record of the day in question indicates that it was unseasonably warm and that the temperature never fell lower than 48 degrees Fahrenheit, 16 degrees above the freezing mark. The plaintiff offers into evidence the man's barograph record of the temperature on January 16.

Is the barograph record admissible?</p>

A

<p>The barograph record is admissible only if it was properly authenticated. Before a writing or any secondary evidence of its content may be received in evidence, the writing must be authenticated by proof showing that the writing is what the proponent says it is. In general, a writing may be authenticated by any evidence that serves to establish its authenticity. One means of authentication under Federal Rule 901(b) is by evidence describing a process or system used to produce a result and showing that the process or system produces an accurate result. Hence, for the man's barograph record to be admissible, evidence must have been offered that the instrument is accurate and that it was in good working order when the record was produced.</p>

81
Q

<p>A professional painter and his apprentice, in business as a partnership, were hired to paint a store. Midway through the job they ran out of paint, so the painter lent his truck to the apprentice to pick up more. On his way to pick up the paint, the apprentice stopped at a post office along the way to mail a personal letter. On pulling out of the post office parking lot, he negligently ran into a parked car, causing extensive damage.

If the car owner brings a negligence action against the painter, will she prevail?

A No, because the apprentice is an independent contractor.

B No, because a bailor is not vicariously liable for the torts of his bailee.

C Yes, because the apprentice's stop at the post office was not a frolic.

D Yes, because the apprentice was using the painter's truck.</p>

A

<p>The car owner can recover because the apprentice was acting within the scope of the partnership business when he drove negligently. Vicarious liability for the conduct of another can arise in partnership and joint venture situations. Each member of the partnership is vicariously liable for the tortious conduct of another partner committed in the scope of the partnership's affairs. As with respondeat superior situations, if the tortfeasor has gone off on a frolic of his own, he is no longer acting within the scope of the partnership and the other partners will not be liable. On the other hand, a minor deviation from the partnership activity will not take it outside of the scope of the partnership's affairs. Here, the painter and the apprentice are partners in their painting business. The apprentice's detour to a post office along the way simply to mail a letter did not take his activity outside the scope of the partnership's affairs; hence the painter is vicariously liable simply because of his status as a partner. (A) is incorrect because the apprentice's status as an independent contractor is irrelevant to this question. While the apprentice is, with respect to the person who hired him, an independent contractor, the apprentice is a partner rather than an independent contractor as to the painter. Because this question concerns the car owner and the painter, the apprentice's status as to the person who hired him is irrelevant and would not save the painter from liability</p>

82
Q

<p>An employee properly sued her employer for wrongful discharge in federal court. During discovery, the employee served the employer with a discovery request for information regarding all employment termination over the previous 15 years, regardless of the position. The employer objected, and the employee filed a motion to compel the requested discovery. The court denied the motion to compel, and the employee wants to file an immediate appeal to review this decision.

Does the employee have a right to an immediate appeal?

A Yes, because the trial court's order is a "collateral order" that is immediately appealable under the collateral order rule.

B Yes, because the trial court's order is a "final judgment" that is immediately appealable.

C Yes, because the plaintiff may obtain immediate appellate review of the trial court's order through a writ of mandamus.

D No, because the trial court's order is an "interlocutory order" that can be reviewed prior to final judgment only if the trial court certifies it for immediate appeal and the appellate court, in its discretion, agrees to hear the appeal.</p>

A

<p>The employee does not have the right to an immediate appeal. Interlocutory orders are the rulings that trial judges make during the course of pretrial proceedings and trials that do not completely resolve the case. Therefore, they are not final. As a result, interlocutory orders are typically not immediately reviewable on appeal until a final order is made, unless they meet one of the exceptions permitting an appeal as of right (i.e., orders granting injunctions; orders appointing a receiver; orders in admiralty cases finding liability but leaving damages to be assessed later; patent infringement orders where only an accounting is ordered; and orders affecting or changing possession of property). The Interlocutory Appeals Act also may permit a review of an interlocutory order, but it is discretionary, and may be available only when (i) the trial judge certifies that the interlocutory order involves a controlling question of law, as to which there is substantial ground for difference of opinion, and immediate appeal from the order may materially advance the ultimate termination of the litigation; and (ii) the court of appeals then agrees to allow the appeal. A party obtaining such a certificate from the trial judge must, within 10 days, apply to the court of appeals, where two out of three judges must agree to hear the appeal. Here, the trial court's denial of the employee's pretrial motion to compel discovery is an interlocutory order. As such, there is no immediate right to appeal. Choice (D) correctly states the rule under the Interlocutory Appeals Act. The Act only offers a discretionary review; it is not reviewable by right. (A) is wrong because it incorrectly classifies the order as a collateral order. The collateral order rule may permit a review if the claim or issue is separable from and collateral to the main suit and is too important to require deferring appellate review. The order then may be classified as a judgment in a separate ("collateral") proceeding. Here, the order relates to a discovery request involving the main issue-wrongful discharge. Therefore, it is not a collateral order</p>

83
Q

<p>A citizen of State A sued a local city newspaper for defamation in state court. The newspaper's defense is that the statements related to the citizen were protected speech under the First Amendment to the United States Constitution.

Which of the following statements regarding federal court subject matter jurisdiction is correct?

A The case cannot be removed or appealed to federal court, including the Supreme Court, because the federal issue in the case is not part of the citizen's well-pleaded complaint.

B The newspaper can remove the case to federal district court, because there is an embedded federal issue that must be determined for the citizen's claim to be resolved, and this creates federal question jurisdiction.

C The case must be litigated through the state court system, but once a decision is rendered by the highest state appellate court, the United States Supreme Court would have subject matter jurisdiction over an appeal concerning the First Amendment issue.

D The citizen can remove the action to the federal district court, because the newspaper's defense has raised a federal question that establishes federal court jurisdiction.</p>

A

<p>The case must be litigated through the state court system. Subject matter jurisdiction in federal court based on a federal question must appear as part of the plaintiff's cause of action as set out in a well-pleaded complaint. The defendant's answer or defense is not relevant because the existence of a defense based on federal law will not give federal question jurisdiction. Supreme Court jurisdiction extends to final judgments rendered by the highest court of a state in which a decision could be had. These judgments may be reviewed by the Supreme Court by certiorari where a right is claimed under the federal Constitution. Here, choice (C) essentially reflects these rules because the defamation case is based on a state law claim, and the citizen did not assert a claim arising under federal question jurisdiction. Therefore, the case must be litigated through the state court system; however, the Supreme Court would have subject matter jurisdiction over an appeal from the highest state court in State A concerning the First Amendment issue</p>

84
Q

<p>Congress enacted a statute appropriating money to the states on condition that the states use the money to support "public performances of classical ballet open to the public." The statute provided that the money was not to be used to support any other type of dance, and that tickets to any performance paid for with these funds were to be distributed to the public on a first come, first served basis.

A state that accepted a grant of $500,000 under the federal statute gave half of the grant to a state-sponsored ballet company. The company had been started 20 years earlier as part of a state effort to bring culture to poor, inner-city areas. By state law enacted when the company was formed, no less than 35% of the tickets to each performance of the ballet company must be distributed to the inner-city school systems to be given to minority school children.

Is the state's method of distributing tickets to the state ballet company's performances constitutional?

A Yes, because the state ballet company is state-operated and the doctrine of federalism prohibits the federal government from directly interfering with state operations.

B Yes, because the state ballet ticket distribution system substantially conforms with the underlying purpose of the federal ticket distribution scheme.

C No, because the state distribution system violates the Supremacy Clause.

D No, because the state distribution system violates equal protection.</p>

A

<p>The state ticket distribution system is unconstitutional because of the Supremacy Clause. A valid act of Congress supersedes any state or local action that conflicts with it. The act here is valid because Congress has the power to spend for the general welfare, and in so doing may place conditions on grants as it sees fit. The state law directly conflicts with the federal law because the federal law requires that tickets be distributed on a first come, first served basis, and the state law requires that 35% of the tickets be given to minority school children. Because the state law conflicts with the federal law, it is invalid. (A), based on the Tenth Amendment, is incorrect because, even if Congress lacks the power to directly regulate the distribution of the tickets in question, the regulation here would still be valid as a spending power condition. The Supreme Court has held that Congress may condition grants under the spending power even where it cannot directly regulate, as long as the conditions are (i) clearly stated, (ii) related to the purpose of the program, and (iii) not unduly coercive. [See South Dakota v. Dole (1987)-conditioning federal highway grants on prohibiting minors from drinking]

(D) is incorrect because the state program probably is valid under the Equal Protection Clause. State programs that favor racial and ethnic minorities are subject to the same strict scrutiny standards as programs that discriminate against minorities: They must be narrowly tailored to promote a compelling government interest. There is a compelling government interest in remedying past discrimination, and the facts indicate that the ballet company was established to remedy the prior lack of cultural opportunities that existed in the inner city. The program also appears to be narrowly tailored, and so would likely survive an equal protection challenge.</p>

85
Q

<p>An elderly woman regularly corresponded with her only niece (her sister's daughter), who lived out of town. One day she sent her niece a letter telling her that she planned to leave everything she owned to the niece upon her death. When the woman died, her will left her entire estate valued at $200,000 to her nephew (her niece's only brother). The nephew wrote his sister a letter telling her that he felt bad about being the only person named in their aunt's will, and added, "I'm going to share her estate with you. We can discuss the details at the funeral."

The niece spent $800 on a round-trip ticket to attend her aunt's funeral. After the funeral, she spoke with her brother, who told her that he had changed his mind about sharing their aunt's estate with her. He went on to say that he would be willing to share the estate with her if she were willing to share their mother's estate with him when their mother passed on. The niece responded by telling him that their mother had already signed over all the property to her. He replied that, given her attitude, he would keep their aunt's estate for himself.

Later, after the two had returned to their respective homes, no longer on speaking terms, the niece sued the nephew for a 50% share of their aunt's estate.

What amount, if any, should the niece realize from her suit?

A Nothing, because the aunt's will left everything to the nephew, and the nephew's letter is an insufficient basis to compel him to share.

B $800, because this represents the niece's actual expenses incurred in reliance on the nephew's letter.

C $100,000 (half of the aunt's estate), because the nephew promised her that in a signed writing.

D $100,000, but only if she shares their mother's estate with the nephew.</p>

A

<p>The niece should not recover in her suit because there is no enforceable promise (i.e., no contract) between the nephew and the niece. Generally, a contract will not be enforced unless consideration has passed between the parties. Consideration is defined as a bargained-for exchange of a benefit to the promisor or a detriment to the promisee. Here, the nephew offered the niece half of their aunt's estate, which is certainly a detriment to him, but the niece offered nothing in return. While the nephew told the niece that they would discuss the details at the funeral, the niece's purchasing a ticket to attend the funeral is not a bargained-for detriment to her (i.e., it is not the price of the exchange) because it does not appear that the nephew's motive for the promise was to induce the niece to come to the funeral. Rather, the nephew's offer was simply to make a gift. Thus, his offer was not an enforceable promise. (B) is incorrect because the niece would be able to recover her reliance damages only under a promissory estoppel theory, and there are no grounds for promissory estoppel here. Under promissory estoppel, a promise is enforceable, at least to the extent necessary to prevent injustice, even though there was no consideration for it, if the promisor should reasonably expect to induce action by his promise and that action is in fact induced. Here, the nephew did not promise to give the niece half of their aunt's estate if she came to the funeral; rather, he only promised to share the estate and said they would talk about it at their aunt's funeral. It is not reasonably foreseeable that, based on the promise, the niece would make a special trip to attend the funeral. Indeed, it is not clear that the niece was induced to go to the funeral by the promise; she may have been planning to attend in any case.</p>

86
Q

<p>In order to get some quick cash to pay off a gambling debt, an acquaintance of the defendant asked him to pretend to break into the acquaintance's home, take some silverware, and return the silverware to the acquaintance. The acquaintance believed that he could both collect the insurance proceeds for the "theft" of the silverware, and sell the silverware on the black market. The acquaintance provided the defendant with his address: "46 Maple Avenue." However, due to a strong windstorm, the house number "9" for "49 Maple Avenue" became detached and slid out of position, making it look like "46 Maple Avenue." Thinking that "49 Maple Avenue" was the acquaintance's home due to the mispositioned house number, the defendant slid open a window that was slightly ajar, entered, and took some silverware he found in the kitchen. The silent burglar alarm alerted the police, who arrived and arrested the defendant a short time later.

Why would a court find the defendant not guilty of burglary?

A He acted under a mistake of law.

B There was no breaking.

C There was no entry.

D He reasonably thought that he was in the acquaintance's home.</p>

A

<p>The court would find the defendant not guilty because he reasonably thought that he was in the acquaintance's home. Common law burglary is the breaking and entering of the dwelling house of another in the nighttime with the intent to commit a felony or larceny inside the house. However, in addition to the specific intent to commit a felony, the defendant also must have intended to break and enter the dwelling. If the defendant reasonably believed it was his acquaintance's home, he would not have the intent to break and enter, as he would believe he had permission to break into and enter the house. (A) is wrong because mistake of law generally is not a defense. Furthermore, the defendant did not appear to be acting under the impression that his acts were lawful. Thus, mistake of law is not an issue in the question. (B) is wrong; opening the window, even though it was ajar, would be considered a breaking under the better view. (C) is wrong; there clearly was an entry into the house.</p>